Endocrine Practice Questions (N2 Exam 2)

¡Supera tus tareas y exámenes ahora con Quizwiz!

Greer Bell is a​ 59-year-old man who was admitted to the hospital with complaints of chest​ pain, dyspnea,​ polyuria, polydipsia, and polyphagia. His glycosylated hemoglobin​ (A1C) is​ 9%. While taking Mr.​ Greer's admission​ history, he mentions that his feet feel numb. Which of the following complications would you assess Mr. Greer for​ first? a. Retinopathy b. Hepatitis c. A foot ulcer d. Nephropathy

c. A foot ulcer

What acute complication will the nurse monitor for in a client with type 2 diabetes​ mellitus? a. Diabetic ketoacidosis b. Atherosclerosis c. Hyperosmolar hyperglycemic state d. Neuropathy

c. Hyperosmolar hyperglycemic state

Assuming the patient eats breakfast at 8:30 AM, lunch at noon, and dinner at 6:00 PM, he or she is at highest risk of hypoglycemia following an 8:00 AM dose of NPH insulin at what time? A. 10:00 AM B. 2:00 PM C. 5:00 PM D. 8:00 PM

C. 5:00 PM

The nurse is preparing to administer insulin to an underweight patient. Which actions should the nurse take when providing this​ injection? Select all that apply. a. Make sure no air bubbles are present in the syringe. b. Insert the needle at a​ 90-degree angle. c. Massage the site of insertion. d. Rotate injection sites. e. Ensure insulin is at room temperature.

a. Make sure no air bubbles are present in the syringe. d. Rotate injection sites. e. Ensure insulin is at room temperature.

Assessing a patient's knowledge of his or her thyroid replacement therapy would show good understanding if he patient stated: a. "My wife may use some of my drug, since she wants to lose weight." b. "I should only need this drug for about 3 months." c. "I can stop taking this drug as soon as I feel like my old self." d. "I should call if I experience unusual sweating, weight gain, or chills and fever."

D

The nurse is reinforcing teaching with a client who has been prescribed levothyroxine (Synthroid) to treat hypothyroidism. Which of the following should the nurse include in the teaching? Select all that apply. A. Weight gain is expected while taking this medication. B. Medication should not be discontinued without the advice of the provider. C. Follow up serum TSH levels should be obtained. D. Take the medication on an empty stomach. E. Use fiber laxatives for constipation.

B, C, D The provider carefully titrate the dosage of this medication. It should be increased to slowly until the client reaches an euthyroid state. Therefore the client should not discontinue the medication unless directed by PCP.

The nurse is providing information to a teenager newly diagnosed with diabetes and his parents. The nurse teaches them that the signs of diabetic ketoacidosis (DKA) include: Standard Text: Select all that apply. 1. Change in mental status. 2. Tachycardia. 3. Fruity breath odor. 4. Rapid, shallow respirations. 5. Abdominal pain.

Correct Answer: 1,3,5 Rationale 1: A change in mental state can be associated with DKA. Rationale 2: Tachycardia is not a typical symptom of DKA. Rationale 3: A fruity breath odor is common when the client is in a state of ketoacidosis. Rationale 4: Respirations are rapid, but deep (Kussmaul's breathing) in DKA. Rationale 5: Abdominal pain is commonly seen with DKA.

The nurse is caring for a child just admitted with diabetic ketoacidosis (DKA). Which of the physician's orders should the nurse question? 1. Neurological checks hourly 2. Insert urinary catheter and measure output hourly. 3. NPH insulin IV at 0.1 units/kg per hour 4. Stat serum electrolytes

Correct Answer: 3 Rationale 1: Hourly neuro checks are an appropriate order. Rationale 2: Urinary catheter and hourly outputs are appropriate. Rationale 3: NPH insulin is never administered IV. A short-acting insulin needs to be ordered. Rationale 4: Stat electrolytes are an appropriate order.

A teenager has arrived in the emergency department (ED) with confusion. The physician suspects diabetic ketoacidosis (DKA). A stat serum glucose is done, and the result is 76l5 mg/dL. The nurse expects that this teen has which symptoms? 1. Tachycardia, dehydration, and abdominal pain 2. Sweating, photophobia, and tremors 3. Dry mucous membranes, blurred vision, and weakness 4. Dry skin, shallow rapid breathing, and dehydration

Correct Answer: 3 Rationale 1: Tachycardia is seen in hypoglycemia. Rationale 2: Sweating, photophobia, and tremors are indicative of hypoglycemia. Rationale 3: Dry mucous membranes, blurred vision, and weakness are seen with hyperglycemia. Rationale 4: Dry skin and dehydration are signs of hyperglycemia, but shallow breathing is a sign of hypoglycemia.

Excessive thirst and volume of very dilute urine may be symptoms of: A. Urinary tract infection B. Diabetes insipidus C. Viral gastroenteritis D.Hypoglycemia

Correct answer: B Diabetes insipidus is a condition in which the kidneys are unable to conserve water, often because there is insufficient antidiuretic hormone (ADH) or the kidneys are unable to respond to ADH. Although diabetes mellitus may present with similar symptoms, the disorders are different. Diabetes insipidus does not involve hyperglycemia.

Polydipsia and poly uria related to diabetes mellitus are primarily due to: a.The release of ketones from cells during fat metabolism b. Fluid shifts resulting from exposure to high levels of hyperglycemia c. Damage to the kidneys from exposure to high levels of glucose d. changes in RBCs resulting from attachemnt of excessibe glucose to hemoglobin

Correct answer: d. Rationale: The osmotic effect of glucose produces the manifesatiaions of polydispsia and polyuria.

What insulin type can be given by IV? Select all that apply: A. Glipizide (Glucotrol) B. Lispro (Humalog) C. NPH insulin D. Glargine (Lantus) E. Regular insulin

E) Regular insulinThe only insulin that can be given by IV is regular insulin.

Cardiac effects of hyperthyroidism include which of the following? a) Decreased pulse pressure b) Bradycardia c) Palpitations d) Decreased systolic blood pressure

Palpitations Explanation: Cardiac effects may include sinus tachycardia, increased pulse pressure, and palpitations. Systolic blood pressure is elevated.

Which of the following are features of type 2 diabetes​ mellitus? Select all that apply. a. Inadequate insulin production b. Ketone production c. Complete insulin deficiency d. Complete destruction of beta cells e. Insulin resistance

a. Inadequate insulin production e. Insulin resistance

A patient with type 1 diabetes mellitus has a serum hematocrit level of​ 24%. What additional finding should the nurse report to the health care​ provider? a. capillary blood glucose of 60​ mg/dL b. the presence of glucose in urine c. glycosylated hemoglobin of 7.0 d. the presence of albumin in urine

a. capillary blood glucose of 60​ mg/dL

The nurse is reviewing data collected for a​ patient's health history. Which factor should the nurse identify as increasing the​ patient's risk of developing type 2 diabetes​ mellitus? a. physical inactivity b. low​ waist-to-hip ratio c. blood pressure of​ 120/70 d. body mass index of 23 ​kg/m2

a. physical inactivity

A nurse performs a physical assessment on a client with type 2 diabetes mellitus. Findings include a fasting blood glucose of 120 mg/dL, temp of 101 F, pulse of 88 bpm, respirations of 22, and blood pressure of 100/72. Which finding would be of most concern to the nurse? 1. Pulse 2. Respiration 3. Temperature 4. Blood pressure

3) temp. An elevated temperature may indicate infection. Infection is a leading cause of hyperglycemic hyperosmolar nonketotic syndrome or diabetic ketoacidosis. The other findings noted in the question are within normal limits.

A client with hyperthyroidism has been given methimazole (a antithyroid med). Which nursing considerations are associated with this medication? SELECT ALL THAT APPLY a. Administer with food b. put client on low-calorie, low-protein diet c. assess for unexplained bruising or bleeding d. instruct client to report SE such as sore throat, fever and HAs e. use special radioactive precautions when handling the clients urine for the first 24 hours following initial administration

A,C, D

The patient is prescribed 30 units regular insulin and 70 units NPH insulin subcutaneously every morning. The nurse will provide which instruction to the patient? A. "Draw up the regular insulin into the syringe first, followed by the cloudy NPH insulin." B. "Mixing insulins will help increase insulin production." C. "Rotate sites at least once weekly." D. "Use a 23- to 25-gauge syringe with a 1-inch needle for maximum absorption."

A. "Draw up the regular insulin into the syringe first, followed by the cloudy NPH insulin."

Which information should be included in a teaching plan for patients taking oral hypoglycemic drugs? (Select all that apply.) A. Limit your alcohol consumption. B. Report symptoms of anorexia and fatigue. C. Take your medication only as needed. D. Notify your physician if blood glucose levels rise above the level set for you.

A. Limit your alcohol consumption. B. Report symptoms of anorexia and fatigue. D. Notify your physician if blood glucose levels rise above the level set for you.

A patient with type 1 diabetes has an unusually high morning glucose measurement, and the health care provider wants the patient evaluated for possible Somogyi effect. The nurse will plan to a. administer an increased dose of NPH insulin in the evening. b. obtain the patient's blood glucose at 3:00 in the morning. c. withhold the nighttime snack and check the glucose at 6:00 AM. d. check the patient for symptoms of hypoglycemia at 2:00 to 4:00 AM.

B Rationale: In the Somogyi effect, the patient's blood glucose drops in the early morning hours (in response to excess insulin administration), which causes the release of hormones that result in a rebound hyperglycemia. It is important to check the blood glucose in the early morning hours to detect the initial hypoglycemia. An increased evening NPH dose or holding the nighttime snack will further increase the risk for early morning hypoglycemia. Information about symptoms of hypoglycemia will not be as accurate as checking the patient's blood glucose in determining whether the patient has the Somogyi effect.

A patient with type 1 diabetes who uses glargine (Lantus) and lispro (Humalog) insulin develops a sore throat, cough, and fever. When the patient calls the clinic to report the symptoms and a blood glucose level of 210 mg/dl, the nurse advises the patient to a. use only the lispro insulin until the symptoms of infection are resolved. b. monitor blood glucose every 4 hours and notify the clinic if it continues to rise. c. decrease intake of carbohydrates until glycosylated hemoglobin is less than 7%. d. limit intake to non-calorie-containing liquids until the glucose is within the usual range.

B Rationale: Infection and other stressors increase blood glucose levels and the patient will need to test blood glucose frequently, treat elevations appropriately with insulin, and call the health care provider if glucose levels continue to be elevated. Discontinuing the glargine will contribute to hyperglycemia and may lead to DKA. Decreasing carbohydrate or caloric intake is not appropriate as the patient will need more calories when ill. Glycosylated hemoglobins are not used to test for short-term alterations in blood glucose.

During a diabetes screening program, a patient tells the nurse, "My mother died of complications of type 2 diabetes. Can I inherit diabetes?" The nurse explains that a.) as long as the patient maintains normal weight and exercises, type 2 diabetes can be prevented. b.) the patient is at a higher than normal risk for type 2 diabetes and should have periodic blood glucose level testing. c.) there is a greater risk for children developing type 2 diabetes when the father has type 2 diabetes. d.) although there is a tendency for children of people with type 2 diabetes to develop diabetes, the risk is higher for those with type 1 diabetes.

B Rationale: Offspring of people with type 2 diabetes are at higher risk for developing type 2 diabetes. The risk can be decreased, but not prevented, by maintenance of normal weight and exercising. The risk for children of a person with type 1 diabetes to develop diabetes is higher when it is the father who has the disease. Offspring of people with type 2 diabetes are more likely to develop diabetes than offspring of those with type 1 diabetes.

A diabetic patient is started on intensive insulin therapy. The nurse will plan to teach the patient about mealtime coverage using _____ insulin. a. NPH b. lispro c. detemir d. glargine

B Rationale: Rapid or short acting insulin is used for mealtime coverage for patients receiving intensive insulin therapy. NPH, glargine, or detemir will be used as the basal insulin.

Which of these laboratory values noted by the nurse when reviewing the chart of a diabetic patient indicates the need for further assessment of the patient? a. Fasting blood glucose of 130 mg/dl b. Noon blood glucose of 52 mg/dl c. Glycosylated hemoglobin of 6.9% d. Hemoglobin A1C of 5.8%

B Rationale: The nurse should assess the patient with a blood glucose level of 52 mg/dl for symptoms of hypoglycemia, and give the patient some carbohydrate-containing beverage such as orange juice. The other values are within an acceptable range for a diabetic patient.

A patient with type 2 diabetes is scheduled for an outpatient coronary arteriogram. Which information obtained by the nurse when admitting the patient indicates a need for a change in the patient's regimen? a. The patient's most recent hemoglobin A1C was 6%. b. The patient takes metformin (Glucophage) every morning. c. The patient uses captopril (Capoten) for hypertension. d. The patient's admission blood glucose is 128 mg/dl.

B Rationale: To avoid lactic acidosis, metformin should not be used for 48 hours after IV contrast media are administered. The other patient data indicate that the patient is managing the diabetes appropriately.

A client's blood glucose level is 45 mg/dl. The nurse should be alert for which signs and symptoms? a) Coma, anxiety, confusion, headache, and cool, moist skin b) Kussmaul's respirations, dry skin, hypotension, and bradycardia c) Polyuria, polydipsia, polyphagia, and weight loss d) Polyuria, polydipsia, hypotension, and hypernatremia

Coma, anxiety, confusion, headache, and cool, moist skin Explanation: Signs and symptoms of hypoglycemia (indicated by a blood glucose level of 45 mf/dl) include anxiety, restlessness, headache, irritability, confusion, diaphoresis, cool skin, tremors, coma, and seizures. Kussmaul's respirations, dry skin, hypotension, and bradycardia are signs of diabetic ketoacidosis. Excessive thirst, hunger, hypotension, and hypernatremia are symptoms of diabetes insipidus. Polyuria, polydipsia, polyphagia, and weight loss are classic signs and symptoms of diabetes mellitus.

The child was diagnosed with phenylketonuria shortly after birth and has been treated by the endocrine clinic for the last four years. The mother has missed the last three appointments. When the child keeps the next appointment, the mother assures the nurse that the child has followed the dietary restrictions. Which finding would make the nurse question this statement? 1. The child's body has a musty odor. 2. This child is a blue-eyed blond. 3. The child appears sleepy and uninterested in the surroundings. 4. The child has a sunburn over his entire body.

Correct Answer: 1 Rationale 1: The odor is caused by the excretion of phenylketone by-products through the skin and would indicate noncompliance with the dietary restrictions. Rationale 2: While this is a characteristic of most children with PKU, it is not related to dietary intake or restrictions. Rationale 3: This is not a symptom of untreated phenylketonuria. Rationale 4: Photophobia is not associated with PKU. The child with untreated PKU has an eczema rash.

The school nurse has noticed an increase in the number of children in the school being diagnosed with type 2 diabetes. Which changes could the nurse implement at school to help reduce students' risk for developing type 2 diabetes? Standard Text: Select all that apply. 1. Increase the amount of daily physical activity. 2. Meet with all parents and explain the risk that is associated with obesity. 3. Test each child's urine monthly. 4. Teach the parents to avoid administering aspirin to their children. 5. Work with the cafeteria to decrease the amount of fat in the foods served.

Correct Answer: 1,2,5 Rationale 1: Increased physical activity will decrease a child's risk of developing type 2 diabetes. Rationale 2: Obese children have an increased risk of type 2 diabetes. Working with the parents, the nurse can reduce the obesity in the school. Rationale 3: Testing urine will not decrease the risk of developing type 2 diabetes, although it may lead to earlier diagnosis of the disease. Rationale 4: Aspirin administration is not related to type 2 diabetes. Rationale 5: A diet high in fat is associated with type 2 diabetes.

The nurse is teaching the caregiver of a child who is newly diagnosed with type 1 diabetes mellitus how to minimize pain with insulin injections. Which interventions to minimize pain will the nurse include in the teaching? Standard Text: Select all that apply. 1. Do not reuse needles. 2. Remove all bubbles from the syringe before injecting. 3. Have the child flex the muscle during injection. 4. Inject insulin when it is cold. 5. Do not change the direction of the needle during insertion or withdrawal.

Correct Answer: 1,2,5 Rationale 1: Reusing needles leads to more pain on injection. Rationale 2: Removing bubbles from the syringe minimizes pain. Rationale 3: Flexing or tensing muscles during injection causes more discomfort. Rationale 4: Insulin should be injected when it is at room temperature to minimize pain. Rationale 5: Keeping the direction of the syringe constant will minimize pain.

Based on physical findings, including a webbed neck and low hairline, the newborn female infant is suspected of having Turner's syndrome. The baby is in the newborn nursery while preparations are made for further evaluation including karyotyping. The nurse will want to monitor this baby for common associated conditions including: 1. Club foot (talipes equinovarus). 2. Congenital heart anomalies. 3. Hyperbilirubinemia due to liver abnormalities. 4. Diaphragmatic hernia.

Correct Answer: 2 Rationale 1: Club foot is not associated with Turner's syndrome. Rationale 2: Congenital heart anomalies, including coarctation of the aorta, frequently are associated with Turner's syndrome. Rationale 3: The girl with Turner's syndrome has the normal risk for hyperbilirubinemia. It is not associated with Turner's syndrome. Rationale 4: Diaphragmatic hernias are not associated with Turner's syndrome.

Mandatory testing in the newborn nursery determines that the infant has hypothyroidism. When discussing the treatment with the new mother, the mother states that she doesn't believe in taking medications. The nurse would explain that failure to treat the infant with the appropriate medication will result in: 1. Heart disease. 2. Mental retardation. 3. Renal failure. 4. Thyroid storm.

Correct Answer: 2 Rationale 1: If the hypothyroidism is left untreated, the child will experience bradycardia but will not develop heart disease. Rationale 2: Untreated hypothyroidism will lead to mental retardation. Rationale 3: Untreated hypothyroidism does not lead to renal failure. Rationale 4: Thyroid storm is a complication of hyperthyroidism, not hypothyroidism.

A patient has been recently diagnosed with type 1 diabetes mellitus. The patient states, "I am thin and eat all the time. How can I have diabetes?" Which response by the nurse is most appropriate? 1. "Thin people can be diabetic, too." 2. "Your condition makes it impossible for you to gain weight." 3. "People with type 1 diabetes are usually thin or of normal weight at diagnosis." 4. "Your lab tests indicate the presence of diabetes."

Correct Answer: 3 Rationale 1 : Although this statement is correct, it does not answer the patient's question. Rationale 2 : It is not impossible for diabetics to gain weight. Rationale 3 : The diabetic patient is unable to obtain the needed glucose for the body's cells, due to the lack of insulin. Patients diagnosed with type 1 diabetes mellitus experience polyphagia and are often thin.

During the hospital stay in the newborn nursery, the infant is tested for galactosemia. When the test is positive, the parents are educated about treatment for galactosemia. The infant will be placed on what type of infant feeding? 1. Goat's milk formula 2. Breast milk 3. Cow's milk-based formula 4. Meat-based formula such as Nutramigen

Correct Answer: 4 Rationale 1: Goat's milk formula contains galactose and is excluded from the infant's diet. Rationale 2: Breast milk contains galactose and is excluded from the infant's diet. Rationale 3: Cow's milk-based formula contains galactose and is excluded from the infant's diet. Rationale 4: The meat-based formula does not contain galactose and is appropriate for the infant's diet.

A client with DKA is being treated in the ED. What would the nurse suspect? 1. Comatose state 2. Decreased Urine Output 3. Increased respirations and an increase in pH. 4. Elevated blood glucose level and low plasma bicarbonate level.

Correct Answer: 4 Rationale: In DKA the arteriole pH is lower than 7.35, plasma bicarbonate is lower than 15 mEq/L, the blood glucose is higher than 250, and ketones are present in the blood and urine. The client would be experiencing polyuria and Kussmauls respirations would be present. A comatose state may occur if DKA is not treated.

A client with diabetes melllitus has a blood glucose of 644mg/dl. The nurse intreprets that this client is most at risk of developing which type of acid base imbalance? A. Metabolic acidosis B. Metabolic alkalosis C. Respiratory Acidosis D. Respiratory Alkalosis"

Correct Answer: A, Metabolic Acidosis Rationale: DM can lead to metabolic acidosis. When the body does not have sufficient circulating insulin, the blood glucose level rises. At the same time, the cells of the body use all available glucose. The body then breaks down glycogen and fat for fuel. The by-products of fat metabolism are acidotic and can lead to the condition known as diabetic ketoacidosis.

A patient recovering from DKA asks the nurse how acidosis occurs. The best response by the nurse is that a. insufficient insulin leads to cellular starvation, and as cells rupture they release organic acids into the blood. b. when an insulin deficit causes hyperglycemia, then proteins are deaminated by the liver, causing acidic by-products. c. excess glucose in the blood is metabolized by the liver into acetone, which is acidic. d. an insulin deficit promotes metabolism of fat stores, which produces large amounts of acidic ketones.

D Rationale: Ketoacidosis is caused by the breakdown of fat stores when glucose is not available for intracellular metabolism. The other responses are inaccurate.

A college student who has type 1 diabetes normally walks each evening as part of an exercise regimen. The student now plans to take a swimming class every day at 1:00 PM. The clinic nurse teaches the patient to a. delay eating the noon meal until after the swimming class. b. increase the morning dose of neutral protamine Hagedorn (NPH) insulin on days of the swimming class. c. time the morning insulin injection so that the peak occurs while swimming. d. check glucose level before, during, and after swimming.

D Rationale: The change in exercise will affect blood glucose, and the patient will need to monitor glucose carefully to determine the need for changes in diet and insulin administration. Because exercise tends to decrease blood glucose, patients are advised to eat before exercising. Increasing the morning NPH or timing the insulin to peak during exercise may lead to hypoglycemia, especially with the increased exercise.

A patient screened for diabetes at a clinic has a fasting plasma glucose level of 120 mg/dl (6.7 mmol/L). The nurse will plan to teach the patient about a. use of low doses of regular insulin. b. self-monitoring of blood glucose. c. oral hypoglycemic medications. d. maintenance of a healthy weight.

D Rationale: The patient's impaired fasting glucose indicates prediabetes and the patient should be counseled about lifestyle changes to prevent the development of type 2 diabetes. The patient with prediabetes does not require insulin or the oral hypoglycemics for glucose control and does not need to self-monitor blood glucose.

What is the only insulin that can be given intravenously? a) Lantus b) Ultralente c) Regular d) NPH

Regular Explanation: Insulins other than regular are in suspensions that could be harmful if administered IV.

The nurse assisting in the admission of a client with diabetic ketoacidosis will anticipate the physician ordering which of the following types of intravenous solution if the client cannot take any fluids orally? a. 0.45% normal saline solution b. Lactated Ringer's solution c. 0.9 normal saline solution d. 5% dextrose in water (D5W)

a. 0.45% normal saline solution Helps to hydrate patient and keep electrolyte levels balanced

The nurse is caring for a​ 70-year-old client admitted for possible type 2 diabetes mellitus. When obtaining the​ client's history, what conditions are potential indicators for diabetes mellitus in an older​ client? Select all that apply. a. Gastroparesis b. Glaucoma c. Periodontal disease d. Hypertension e. Impotence

a. Gastroparesis b. Glaucoma c. Periodontal disease e. Impotence

Preoperative instructions for the patient scheduled for a subtotal thyroidectomy includes teaching the patient a. how to support the head with the hands when moving b. that coughing should due avoided to prevent pressure on the incision c. that the head and neck will need to remain immobile until the incision heals d. that any tingling around the lips or in the fingers after surgery is expected and temporary

a. how to support the head with the hands when moving (rationale- to prevent strain on the suture line postoperatively, the head must be manually supported while turning and moving in bed, but range-of-motion exercise for the head and neck are also taught preoperatively to be gradually implemented after surgery. There is no contraindication for coughing and deep breathing, and they should be carrier out postoperatively. Tingling around the lips or fingers is a sign of hypocalcemia, which may occur if the parathyroid glands are inadvertently removed during surgery, and should be reported immediately.)

The nurse is trying to determine if a patient is experiencing manifestations of type 1 or type 2 diabetes mellitus. Which question should the nurse ask the patient to help determine the​ type? a. ​"Have you been hungrier than in the​ past?" b. ​"Have you been drinking more liquids than in the​ past?" c. ​"Have you noticed any changes in your​ vision?" d. ​"Have you been urinating in greater amounts than in the​ past?"

a. ​"Have you been hungrier than in the​ past?"

Patients with type 1 diabetes mellitus may require which of the following changes to their daily routine during times of infection? a. no change b. less insulin c. more insulin d. oral diabetic agents

answer C: during times of infection and illness diabetic patients may need even more insulin to compensate for increased blood glucose levels.

A nurse is teaching a group of patients about the prevalence of type 2 diabetes in older adults. The nurse knows teaching has been effective when a patient​ states, "Statistically, in a group of 100 older adults in the United​ States, approximately: a. 17 will have type 2​ diabetes." b. 27 will have type 2​ diabetes." c. 10 will have type 2​ diabetes." d.33 will have type 2​ diabetes."

b. 27 will have type 2​ diabetes."

Which of the following items should be considered when developing a care plan for the older client with diabetes​ mellitus? Select all that apply. a. Increased hunger b. Exercises adjusted for physical limitations c. Excess fluid intake d. Insufficient financial resources to afford medications e. Visual deficits that might interfere with insulin administration

b. Exercises adjusted for physical limitations d. Insufficient financial resources to afford medications e. Visual deficits that might interfere with insulin administration

The nurse is caring for a patient experiencing diabetic ketoacidosis. What actions should the nurse take when preparing this​ patient's insulin​ infusion? Select all that apply. a. Prepare an infusion of Dextrose​ 5% and​ 0.45% normal saline. b. Flush the tubing with the insulin solution before connecting. c. Discontinue the infusion after first dose of subcutaneous insulin. d. Have one ampule of Dextrose​ 10% at the bedside. e. Attach insulin infusion to an intravenous pump.

b. Flush the tubing with the insulin solution before connecting. c. Discontinue the infusion after first dose of subcutaneous insulin. e. Attach insulin infusion to an intravenous pump.

An older patient with type 2 diabetes mellitus is upset because family members do not believe the patient has an illness and resist helping with diet and activity modifications. What should the nurse suggest to help this​ patient? a. Explain the risk for family also to develop the illness. b. Invite family to participate in a support group. c. Limit discussions about the illness with family members. d. Store​ health-related items away from common family areas in the home.

b. Invite family to participate in a support group.

Which of the following are risk factors associated with type 2 diabetes​ mellitus? Select all that apply. a. Weight loss b. Physical inactivity c. Blood pressure greater than or equal to​ 130/85 mmHg d. HDL cholesterol greater than or equal to 50​ mg/dl e. Triglyceride level greater than or equal to 250​ mg/dl

b. Physical inactivity c. Blood pressure greater than or equal to​ 130/85 mmHg e. Triglyceride level greater than or equal to 250​ mg/dl

The nurse is reviewing the healthcare record of a patient with type 2 diabetes mellitus who has a foot wound. Which outcome of care would be the most important at this​ time? a. The patient will obtain a thermometer for monitoring bath water temperature. b. The patient will bring a caregiver to the next healthcare appointment. c. The patient will explain why patients with diabetes should not go barefoot. d. The patient will describe the steps of effective diabetic foot care.

b. The patient will bring a caregiver to the next healthcare appointment.

A patient with SIADH is treated with water restriction and administration of IV fluids. The nurses evaluates that treatment has been effective when the patient experiences a. increased urine output, decreased serum sodium, and increased urine specific gravity b. increased urine output, increased serum sodium, and decreased urine specific gravity c. decreased urine output, increased serum sodium, and decreased urine specific gravity d. decreased urine output, decreased serum sodium, and increased urine specific gravity

b. increased urine output, increased serum sodium, and decreased urine specific gravity (rationale- the patient with SIADH has water retention with hyponatremia, decreased urine output and concentrated urine with high specific gravity. improvement in the patient's condition reflected by increased urine output, normalization of serum sodium, and more water in the urine, decreasing the specific gravity.)

A patient with diabetes asks what can be done to prevent the development of corns on the feet. How should the nurse respond to this​ patient? a. ​"Use corn pads to gradually remove the​ growths." b. ​"Make sure that you select shoes that are appropriately​ fitted." ​c. "Corns are best treated by shaving them​ off." d. ​"A mild abrasive soap can be used to scrub the area to remove​ them."

b. ​"Make sure that you select shoes that are appropriately​ fitted."

A patient with DI is treated with nasal desmopression. The nurse recognize that the drug is not having an adequate therapeutic effect the the patient experiences a. headache and weight gain b. nasal irritation and nausea c. a urine specific gravity of 1.002 d. an oral intake greater than urinary output

c. a urine specific gravity of 1.002 (rationale- normal urine specific gravity is 1.003 to 1.030, and urine with a specific gravity of 1.002 is very dilute, indicating that there continues to be excessive loss of water and that treatment of DI is inadequate. H/A, weight gain, and oral intake greater the urinary output are signs of volume excess that occur with overmedication. Nasal irritation & nausea may also indicate overmedication.)

When providing discharge instructions to a patient following a subtotal thyroidectomy, the nurse advises the patient to a. never miss a daily dose of thyroid replacement therapy b. avoid regular exercise until thyroid function is normalized c. avoid eating foods such as soybeans, turnips, and rutabagas d. use warm salt water gargles several times a day to relieve throat pain

c. avoid eating foods such as soybeans, turnips, and rutabagas (Rationale- when a patient has had a subtotal thyroidectomy, thyroid replacement therapy is not given, because exogenous hormone inhibits pituitary production of TSH and delays or prevents the restoration of thyroid tissue regeneration. However, the patient should avoid goitrogens, foods that inhibit thyroid, such as soybeans, turnips, rutabagas, and peanut skins. REgular exercise stimulates the thyroid gland and is encourage. Salt water gargles are used for dryness and irritation of the mouth and throat following radioactive iodine therapy.)

An adolescent client with type I diabetes mellitus is admitted to the emergency department for treatment of diabetic ketoacidosis. Which assessment findings should the nurse expect to note? a) sweating and tremors b) hunger and hypertension c) cold, clammy skin and irritability d) fruity breath and decreasing level of consciousness

d) fruity breath and decreasing level of consciousness"Hyperglycemia occurs with diabetic ketoacidosis. Signs of hyperglycemia include fruity breath and a decreasing level of consciousness. Hunger can be a sign of hypoglycemia or hyperglycemia, but hypertension is not a sign of diabetic ketoacidosis. Instead, hypotension occurs because of a decrease in blood volume related to the dehydrated state that occurs during diabetic ketoacidosis. Cold, clammy skin, irritability, sweating, and tremors are all signs of hypoglycemia."

When taking a health history, the nurse screens for manifestations suggestive of diabetes type I. Which of the following manifestations are considered the primary manifestations of diabetes type I and would be most suggestive of diabetes type I and require follow-up investigation? a. Excessive intake of calories, rapid weight gain, and difficulty losing weight b. Poor circulation, wound healing, and leg ulcers, c. Lack of energy, weight gain, and depression d. An increase in three areas: thirst, intake of fluids, and hunger

d. An increase in three areas: thirst, intake of fluids, and hunger "The primary manifestations of diabetes type I are polyuria (increased urine output), polydipsia (increased thirst), polyphagia (increased hunger). Excessive calorie intake, weight gain, and difficulty losing weight are common risk factors for type 2 diabetes. Poor circulation, wound healing and leg ulcers are signs of chronic diabetes. Lack of energy, weight gain and depression are not necessarily indicative of any type of diabetes."

A patient with diabetes mellitus and poor circulation has thick and ingrown toenails. What should the nurse instruct the patient to​ do? a. Cut toenails immediately prior to bathing. b. Soak feet in Epsom salts daily. c. Use a clean sharp razor blade to trim nails. d. Make an appointment with a podiatrist.

d. Make an appointment with a podiatrist.

Which of the following should the nurse perform when caring for the diabetic​ client's feet? a. Let the feet air dry. b. Cut the toenails as short as possible. c. Remove all corns on the feet with a commercial product. d. Wash the feet with mild soap and lukewarm water.

d. Wash the feet with mild soap and lukewarm water.

Richard​ Cortez, a​ 68-year-old man with history of type 2 diabetes​ mellitus, is admitted to the hospital with​ community-acquired pneumonia. What should you teach Mr. Cortez to help him prevent injuries while he is in the​ hospital? a. Monitor blood glucose levels once a week. b. Turn off lights around the bed and room. c. Walk slowly when feeling dizzy from medications. d. Wear shoes or slippers when out of bed.

d. Wear shoes or slippers when out of bed.

Physical changes of hypothyroidism that must be monitored when replacement therapy is started include a. achlorhydria and constipation b. slowed mental processes and lethargy c. anemia and increased capillary fragility d. decreased cardiac contractility and coronary atherosclerosis

d. decreased cardiac contractility and coronary atherosclerosis (rationale- hypothyroidism affects the heart in many ways, causing cardiomyopathy, coronary atherosclerosis, bradycardia, pericardial effusions, and weakened cardiac contractility. when thyroid replacement therapy is started, myocardial oxygen consumption is increased and the resultant oxygen demand may cause angina, cardiac arrhythmias, and heart failures. It is important to monitor patients with compromised cardiac status when starting replacement therapy.)

Physical changes of hypothyroidism that must be monitored when replacement therapy is started include a. achlorhydria and constipation b. slowed mental processes and lethargy c. anemia and increased capillary fragility d. decreased cardiac contractility and coronary atherosclerosis

d. decreased cardiac contractility and coronary atherosclerosis (rationale- hypothyroidism affects the heart in many ways, causing cardiomyopathy, coronary atherosclerosis, bradycardia, pericardial effusions, and weakened cardiac contractility. when thyroid replacement therapy is started, myocardial oxygen consumption is increased and the resultant oxygen demand may cause angina, cardiac arrhythmias, and heart failures. It is important to monitor patients with compromised cardiac status when starting replacement therapy.)

The community nurse is teaching a group of members with type 1 or 2 diabetes mellitus who are planning to participate in an athletic triathlon. On which potential complication from this event should the nurse focus when teaching? a. impaired glucose tolerance b. diabetic ketoacidosis c. hyperosmolar hyperglycemic state d. hypoglycemia

d. hypoglycemia

The community nurse is teaching a group of members with type 1 or 2 diabetes mellitus who are planning to participate in an athletic triathlon. On which potential complication from this event should the nurse focus when​ teaching? a. impaired glucose tolerance b. diabetic ketoacidosis c. hyperosmolar hyperglycemic state d. hypoglycemia

d. hypoglycemia

A patient with type 1 diabetes has received diet instruction as part of the treatment plan. The nurse determines a need for additional instruction when the patient says, "a. "I may have an occasional alcoholic drink if I include it in my meal plan." b. "I will need a bedtime snack because I take an evening dose of NPH insulin." c. "I will eat meals as scheduled, even if I am not hungry, to prevent hypoglycemia." d. "I may eat whatever I want, as long as I use enough insulin to cover the calories."

"D. "I may eat whatever I want, as long as I use enough insulin to cover the calories." Rationale: Most patients with type 1 diabetes need to plan diet choices very carefully. Patients who are using intensified insulin therapy have considerable flexibility in diet choices but still should restrict dietary intake of items such as fat, protein, and alcohol. The other patient statements are correct and indicate good understanding of the diet instruction."

A nurse is teaching a client with diabetes mellitus about self-management of his condition. The nurse should instruct the client to administer 1 unit of insulin for every: a) 20 g of carbohydrates. b) 15 g of carbohydrates. c) 10 g of carbohydrates. d) 25 g of carbohydrates.

15 g of carbohydrates. Explanation: The nurse should instruct the client to administer 1 unit of insulin for every 15 g of carbohydrates.

The nurse is admitting a client with diabetes mellitus. Which information should the nurse specifically include in the health​ history? Select all that apply. a. Numbness in feet b. Vision changes c. Infections d. Frequent voiding e. Hypotension

a. Numbness in feet b. Vision changes c. Infections d. Frequent voiding

Diabetes mellitus is the leading cause of which of the following​ complications? a. Encephalopathy ​b. End-stage renal disease c. Coronary artery disease d. Heart failure

b. End-stage renal disease

A client is taking glyburide (DiaBeta), 1.25 mg P.O. daily, to treat type 2 diabetes. Which statement indicates the need for further client teaching about managing this disease? a) "I always carry hard candy to eat in case my blood sugar level drops." b) "I avoid exposure to the sun as much as possible." c) "I skip lunch when I don't feel hungry." d) "I always wear my medical identification bracelet."

"I skip lunch when I don't feel hungry." Explanation: The client requires further teaching if he states that he skips meals. A client who is receiving an oral antidiabetic agent should eat meals on a regular schedule because skipping a meal increases the risk of hypoglycemia. Carrying hard candy, avoiding exposure to the sun, and always wearing a medical identification bracelet indicate effective teaching.

A nurse is caring for a cient with type 1 diabetes mellitus. which client complaint would alert the nurse to the presence of a possible hypoglycemic reaction? 1. Tremors 2. Anorexia 3. Hot, dry skin 4. Muscle cramps

1) tremorsdecreased blood glucose levels produce autonomic nervous system symptoms, which are manifested classically as nervousness, irritability, and tremors. option 3 is more likely for hyperglycemia, and options 2 and 4 are unrelated to the signs of hypoglycemia.

The risk factors for type 1 diabetes include all of the following except: a. Diet b. Genetic c. Autoimmune d. Environmental

A: Type 1 diabetes is a primary failure of pancreatic beta cells to produce insulin. It primarily affects children and young adults and is unrelated to diet.

A nurse is preparing a plan of care for a client with diabetes mellitus who has hyerglycemia. The priority nursing diagnosis would be: 1. Deficient knowledge 2. Deficient fluid volume 3. Compromised family coping 4. Imbalanced nutrition less than body requirements

2) deficient fluid volumeAn increased blood glucose level will cause the kidneys to excrete the glucose in the urine. This glucose is accompanied by fluids and electrolytes, causing an osmotic diuresis leading to dehydration. This fluid loss must be replaced when it becomes severe.

The nurse is caring for a client with long-term Type 2 diabetes and is assessing the feet. Which assessment data would warrant immediate intervention by the nurse? 1)The client has crumbling toenails 2)The client has athlete's feet 3)The client has a necrotic big toe 4)The client has thickened toenails.

3) Nectrotic big toe" 1)Crumbling toenails indicate tinea unguium, which is a fungus infection of the toenail. 2)Athlete's foot is a fungal infection that is not life threatening. 3)A necrotic big toe indicates "dead" tissue. The client does not feel pain in the lower extremity and does not realize there has been an injury and therefore does not seek treatment. Increased blood glucose levels decrease oxygen supply that is needed to heal the wound and increase the risk for developing an infection. 4)Big, thick toenails are fungal infections and would not require immediate intervention by the nurse; 50% of the adult population has this.

Which is a rapid-acting insulin with an onset of action of less than 15 minutes? A. insulin glargine (Lantus) B. insulin aspart (NovoLog) C. regular insulin (Humulin R) D. insulin detemir (Levemir)

B. insulin aspart (NovoLog)

Of which of the following symptoms might an older woman with diabetes mellitus complain? = 1) anorexia 2)pain intolerance 3) weight loss 4) perineal itching

4) perineal itching Rationale: Older women might complain of perineal itching due to vaginal candidiasis.

A client with diabetes mellitus demonstratees acute anxiety when first admitted for the treatment of hyperglycemia. The most appropriate intervention to decrease the client's anxiety would be to 1. administer a sedative 2. make sure the client knows all the correct medical terms to understand what is happening 3. ignore the signs and symptoms of anxiety so that they will soon disappear 4. convey empathy, trust, and respect toward the client

4. The most appropriate intervention is to address the client's feelings related to the anxiety

A nurse is interviewing a client with type 2 diabetes mellitus. which statement by the client indicated an understanding of the treatment for this disorder? 1. "i take oral insulin instead of shots" 2. "by taking these medications I am able to eat more" 3. "when I become ill, I need to increase the number of pills I take" 4. "the medications I'm taking help release the insulin I already make"

4.)Clients with type 2 diabetes mellitus have decreased or imparied insulin secretion. Oral hypoglycemic agents are given to these clients to facilitate glucose uptake. Insulin injections may be given during times of stress-induced hyperglycemia. Oral insulin is not available because of the breakdown of the insulin by digestion. Options 1, 2 and 3 are incorrect

A patient with type 1 diabetes has been using self-monitoring of blood glucose (SMBG) as part of diabetes management. During evaluation of the patient's technique of SMBG, the nurse identifies a need for additional teaching when the patient a. chooses a puncture site in the center of the finger pad. b. washes the puncture site using soap and water. c. says the result of 130 mg indicates good blood sugar control. d. hangs the arm down for a minute before puncturing the site.

A Rationale: The patient is taught to choose a puncture site at the side of the finger pad. The other patient actions indicate that teaching has been effective.

Which of the following are common side effects of thiomides? SELECT ALL THAT APPLY a. skin rash b. bradycardia c. diarrhea d. cold e. nausea

A, B, E diarrhea and cold are SE of iodine solution drugs

Prediabetes is associated with all of the following except: a. Increased risk of developing type 2 diabetes b. Impaired glucose tolerance c. Increased risk of heart disease and stroke d. Increased risk of developing type 1 diabetes

ANSWER: D Persons with elevated glucose levels that do not yet meet the criteria for diabetes are considered to have prediabetes and are at increased risk of developing type 2 diabetes. Weight loss and increasing physical activity can help people with prediabetes prevent or postpone the onset of type 2 diabetes.

The nurse caring for a 54-year-old patient hospitalized with diabetes mellitus would look for which of the following laboratory test results to obtain information on the patient's past glucose control? a. prealbumin level b. urine ketone level c. fasting glucose level d. glycosylated hemoglobin level

Answer d: A glycosylated hemoglobin level detects the amount of glucose that is bound to red blood cells (RBCs). When circulating glucose levels are high, glucose attaches to the RBCs and remains there for the life of the blood cell, which is approximately 120 days. Thus the test can give an indication of glycemic control over approximately 2 to 3 months.

Thyroid replacement therapy is indicated for the treatment of a. obesity b. myxedema c. Graves' disease d. acute thyrotoxicosis

B myxedema

One of the benefits of Glargine (Lantus) insulin is its ability to: a.Release insulin rapidly throughout the day to help control basal glucose. b. Release insulin evenly throughout the day and control basal glucose levels. c. Simplify the dosing and better control blood glucose levels during the day. d. Cause hypoglycemia with other manifestation of other adverse reactions.

B) Release insulin evenly throughout the day and control basal glucose levels"Glargine (Lantus) insulin is designed to release insulin evenly throughout the day and control basal glucose levels.

A nurse is assessing a client who has DKA and ketones in the urine. Which of the following are expected findings? Select all that apply. A. Weight gain B. Fruity odor or breath C. Abdominal pain D. Kussmaul respirations E. Metabolic acidosis

B, C, D, E

A nurse is collecting an admission history from a female client who has hypothyroidism. Which of the following findings are expected with this condition? Select all that apply. A. Diarrhea B. Menorrhagia C. Dry skin D. Increased libido E. Hoarseness

B, C, E Abnormal menstrual periods, including menorrhagia and amenorrhea, are clinical manifestations of hypothyroidism. Dry skin and hoarseness are clinical manifestations of hypothyroidism.

A patient is prescribed levothyroxine, thyroid drug. The nurse understands that this drug contains which of the following? a. T3 b. iodine c. T4 d. Vitamin D

C T4

All of the following are common adverse effects of iodine solution drugs except: a. metallic taste b. mouth burning c. cold d. bradycardia e. diarrhea

D bradycardia is seen in thiomides

A hospitalized diabetic patient receives 12 U of regular insulin mixed with 34 U of NPH insulin at 7:00 AM. The patient is away from the nursing unit for diagnostic testing at noon, when lunch trays are distributed. The most appropriate action by the nurse is to a. save the lunch tray to be provided upon the patient's return to the unit. b. call the diagnostic testing area and ask that a 5% dextrose IV be started. c. ensure that the patient drinks a glass of milk or orange juice at noon in the diagnostic testing area. d. request that the patient be returned to the unit to eat lunch if testing will not be completed promptly.

D Rationale: Consistency for mealtimes assists with regulation of blood glucose, so the best option is for the patient to have lunch at the usual time. Waiting to eat until after the procedure is likely to cause hypoglycemia. Administration of an IV solution is unnecessarily invasive for the patient. A glass of milk or juice will keep the patient from becoming hypoglycemic but will cause a rapid rise in blood glucose because of the rapid absorption of the simple carbohydrate in these items.

The patient is receiving propylthiouracil (PTU), a thyomide. The nurse anticipates a reduction in the patients dosage based on assessment of which of the following? a. nervousness b. tachycardia c. weight loss d. decreased appetite

D decreased appetite PTU can cause thyroid suppression leading to s/s of hypothyroidism such as decreased appetite.

The nurse would include which statement when teaching a patient about insulin glargine? A. "You should inject this insulin just before meals because it is very fast acting." B. "The duration of action for this insulin is approximately 8 to10 hours, so you will need to take it twice a day." C. "You can mix this insulin with NPH insulin to enhance its effects." D. "You cannot mix this insulin with regular insulin and thus will have to take two injections."

D. "You cannot mix this insulin with regular insulin and thus will have to take two injections."

A patient with type 1 diabetes mellitus voided​ 4,000 mL of urine in the past 24 hours. The​ patient's skin turgor is​ poor, and the patient is reporting polyphagia and polydipsia. Which blood glucose level should the nurse expect when assessing this​ patient? a. 180​ mg/dL b. 60​ mg/dL c. 125​ mg/dL d. 110​ mg/dL

a. 180​ mg/dL

The nurse is teaching a patient with diabetes about the illness. The nurse knows the teaching has been effective when the patient identifies which statements as being true of pancreatic​ cells? Select all that apply. a. Beta cells secrete insulin. b. Cephalon cells produce creatine. c. Delta cells produce somatostatin. d. Alpha cells produce glucagon. e. Epsilon cells produce erythropoietin.

a. Beta cells secrete insulin. c. Delta cells produce somatostatin. d. Alpha cells produce glucagon.

Walter​ Wariner, an​ 82-year-old man, is complaining of frequent bouts of nausea and indigestion. He also states that he has been experiencing numbness and tingling in his feet. Which of the following statements should you say to Mr.​ Wariner? a. ​"These may be signs of hypertension. You should have your blood pressure​ checked." ​b. "These may be symptoms of diabetes mellitus. You should have your blood sugar​ checked." ​c. "These are normal signs of aging. There is no need to​ worry." ​d. "These may be signs of renal failure. You should have your kidneys​ checked."

b. "These may be symptoms of diabetes mellitus. You should have your blood sugar​ checked."

The nurse is conducting an educational session with a patient who is newly diagnosed with diabetes. The nurse knows further education is needed when the patient​ states, "In the​ U.S.: a. 25.8 million people have​ diabetes." b. 6 million people are diagnosed with diabetes per​ year." c. 7 million people have diabetes but have not been​ diagnosed." d. 18.8 million people have been diagnosed with​ diabetes."

b. 6 million people are diagnosed with diabetes per​ year."

Nutritional recommendations for clients with diabetes mellitus include which of the​ following? ​a. 20% carbohydrates,​ 10% protein, and​ 65% fat b. 65% carbohydrates,​ 20% protein, and​ 10% fat c. ​10% carbohydrates,​ 65% protein, and​ 20% fat ​d. 15% carbohydrates,​ 30% protein, and​ 55% fat

b. 65% carbohydrates,​ 20% protein, and​ 10% fat

A client is seen in the health care​ providers office. What data should alert the nurse as a risk factor associated with the development of type 2 diabetes​ mellitus? a. HDL cholesterol level of 38​ mg/dL b. Blood pressure of​ 140/90 mmHg c. Triglyceride level of 180​ mg/dL d. History of delivering a baby weighing less than 8 lb

b. Blood pressure of​ 140/90 mmHg

The manager observes a graduate nurse teaching a​ 5-year-old patient with diabetes mellitus. The manager determines that content being instructed is appropriate when the nurse​ states, "Insulin acts​ like: a. a mud pie that makes the blood vessels thick and​ sticky." b. a wagon that carries sugar into the cells of the​ body." c. salty potato chips that make people feel very​ thirsty." d. building blocks that help make protein into strong​ muscles."

b. a wagon that carries sugar into the cells of the​ body."

The nurse notes that a​ 41-year-old patient's fasting blood glucose level is 125​ mg/dL. What should the nurse suspect is occurring with the​ patient? a. consistent with diabetes b. consistent with prediabetes c. severe hyperglycemia d. normal results

b. consistent with prediabetes

A patient with type 1 diabetes mellitus who had one episode of vomiting in the past 2 hours asks if the routine insulin injection should be taken. What action by the nurse is best at this​ time? a. Contact the physician. b. Check the​ patient's fasting serum glucose level. c. Explain the need to take the insulin. d. Document the refusal and continue on with the planned care.

c. Explain the need to take the insulin.

Which of the following lab values indicate that the client may have diabetes​ mellitus? a. Casual plasma glucose greater than 150​ mg/dL b. Glycosylated hemoglobin​ (A1C) less than​ 5.7% c. Glycosylated hemoglobin​ (A1C) greater than​ 6.5% d. Fasting plasma glucose less than 126​ mg/dl

c. Glycosylated hemoglobin​ (A1C) greater than​ 6.5%

The nurse is concerned that a patient with type 1 diabetes mellitus is at risk for developing diabetic ketoacidosis. What did the nurse assess to come to this​ conclusion? a. ulcer on plantar aspect of right foot b. reports of anxiety c. serum glucose level of 325​ mg/dL ​d. pale, cool skin

c. serum glucose level of 325​ mg/dL

The nurse is assessing a patient with type 2 diabetes mellitus. What questions should the nurse ask to determine the​ patient's risk for a lower extremity​ amputation? Select all that apply. a. ​"When were you first diagnosed with diabetes​ mellitus?" b. ​"Do you use insulin or oral hypoglycemic​ agents?" c. ​"Do you have any problems with your kidney related to​ diabetes?" d. ​"Do you have any problems with your eyes related to​ diabetes?" e. ​"What were your glycosylated hemoglobin values over the past​ year?"

c. ​"Do you have any problems with your kidney related to​ diabetes?" d. ​"Do you have any problems with your eyes related to​ diabetes?" e. ​"What were your glycosylated hemoglobin values over the past​ year?"

A patient with Grave's dz asks the nurse what caused the disorder. The best response by the nurse is a. "The cause of Grave's disease is not known, although it is thought to be genetic." b. "It is usually associated with goiter formation from an iodine deficiency over a long period of time." c. "Antibodies develop against thyroid tissue and destroy it, causing a deficiency of thyroid hormones" d. "In genetically susceptible persons antibodies form that attack thyroid tissue and stimulate overproduction of thyroid hormones."

d. "In genetically susceptible persons antibodies form that attack thyroid tissue and stimulate overproduction of thyroid hormones." (rationale- The antibodies present in Graves' disease that attack thyroid tissue cause hyperplasia of the gland and stimulate TSH receptors on the thyroid and activate the production of thyroid hormones, creating hyperthyroidism. The disease is not directly genetic, but individuals appear to have a genetic susceptibility to become sensitized to develop autoimmune antibodies. Goiter formation from insufficient iodine intake is usually associated with hypothyroidism.)

A patient recently diagnosed with diabetes wants to check the urine for glucose instead of using capillary blood because of the cost. Which response should the nurse make to the​ patient? a. ​"Urine testing is as reliable as finger stick​ testing." b. ​"Yes, urine testing is cheaper than glucose test​ strips." c. ​"Would you like to switch to this method of​ monitoring?" ​d. "Urine testing is best when combined with serum​ testing."

d. "Urine testing is best when combined with serum​ testing."

Causes of primary hypothyroidism in adults include a. malignant or benign thyroid nodules b. surgical removal or failure of the pituitary gland c. surgical removal or radiation of thyroid gland d. autoimmune-induced atrophy of the gland

d. autoimmune-induced atrophy of the gland (rationale- both Graves disease and Hasimotos thyroiditis are autoimmune disorders that eventually destroy the thyroid gland, leading to primary hypothyroidism. Thyroid tumors most often result in hyperthyroidism. Secondary hypothyroidism occurs as a result of pituitary failure, and iatrogenic hypothyroidism results from thyroidectomy or radiation of the thyroid gland.)

Which statement by the patient demonstrates an understanding of discharge instructions on the use of levothyroxine (Synthroid)? "I will take this medication in the morning so as not to interfere with sleep." "I will double my dose if I gain more than 1 pound per day." "I will stop the medication immediately if I lose more than 2 pounds in a week." "I can expect to see relief of my symptoms within 1 week."

"I will take this medication in the morning so as not to interfere with sleep."

The nurse is caring for a client who has normal glucose levels at bedtime, hypoglycemia at 2am and hyperglycemia in the morning. What is this client likely experiencing? A. Dawn phenomenon B. Somogyi effect C. An insulin spike D. Excessive corticosteroids

1. B The Somogyi effect is when blood sugar drops too low in the morning causing rebound hyperglycemia in the morning. The hypoglycemia at 2am is highly indicative. The Dawn phenomenon is similar but would not have the hypoglycemia at 2am.

"The nurse is teaching a community class to peole with Type 2 diabetes mellitus. Which explanation would explain the development of Type 2 diabetes? 1. The islet cells in the pancreas stop producing insulin. 2. The client eats too many foods that are high in sugar. 3 The pituitary gland does not produce vasopression. 4. The cells become resistant to the circulating insulin.

1. This is the cause of Type 1 diabetes mellitus. 2. This may be a reason for obesity, which may lead to Type 2 diabetes, but eating too much sugar does not cause diabetes. 3. This is the explanation for diabetes insipidus, which should not be confused with diabetes mellitus. 4. (CORRECT) Normally insulin binds to special receptor sites on the cells and initiates a series of reactions involved in metabolism. In Type 2 diabetes these reactions are diminished primarily as a result of obesity and aging.

The thyroid gland produces the thyroid hormones triiodothyronine (T3) and tetraiodothyronine (T4), which are dependent on the availability of a. iodine produced in the liver b. iodine found in the diet c. iron absorbed from the GI tract d. parathyroid hormone (PTH) to promote iodine building

B Iodine in the diet

A 63-year-old patient is newly diagnosed with type 2 diabetes. When developing an education plan, the nurse's first action should be to a. assess the patient's perception of what it means to have type 2 diabetes. b. demonstrate how to check glucose using capillary blood glucose monitoring. c. ask the patient's family to participate in the diabetes education program. d. discuss the need for the patient to actively participate in diabetes management.

A Rationale: Before planning education, the nurse should assess the patient's interest in and ability to self-manage the diabetes. After assessing the patient, the other nursing actions may be appropriate, but planning needs to be individualized to each patient.

A nurse is preparing a teaching plan for a client with diabetes Mellitus regarding proper foot care. Which instruction is included in the plan? 1. Soak feet in hot water 2. apply a moisturizing lotion to dry feet but not between the toes 3. Always have a podiatrist cut your toenails, never cut them yourself 4. avoid using mild soap on the feet

2. The client is instructed to use a moisturizing lotion on the feet and to avoid applying the lotion between the toes.

The nurse is discharging a client diagnosed with diabetes insipidus. Which statementmade by the client warrants further intervention? 1."I will keep a list of my medications in my wallet and wear a Medi bracelet." 2."I should take my medication in the morning and leave it refrigerated at home." 3."I should weigh myself every morning and record any weight gain." 4."If I develop a tightness in my chest, I will call my health-care provider.

2."I should take my medication in the morning and leave it refrigerated at home."" 1.The client should keep a list of medication being taken and wear a Medic Alert bracelet. 2. Medication taken for DI is usually every 8-12 hours, depending on the client. Theclient should keep the medication close at hand. 3.The client is at risk for fluid shifts. Weighing every morning allows the client to follow thefluid shifts. Weight gain could indicate too much medication. 4.Tightness in the chest could be an indicator that the medication is not being tolerated; if this occurs the client should call the health-care provider

A client with type I diabetes is placed on an insulin pump. The most appropriate short-term goal when teaching this client to control the diabetes is: 1) adhere to the medical regimen 2) remain normoglycemic for 3 weeks 3) demonstrate the correct use of the administration equipment. 4) list 3 self care activities that are necessary to control the diabetes"

3.) is correct "1) this is not a short-term goal 2) this is measurable, but it's a long-term goal 3) this is a short-term goal, client oriented, necessary for the client to control the diabetes, and measurable when the client performs a return demonstration for the nurse 4) although this is measurable and a short-term goal, it is not the one with the greatest priority when a client has an insulin pump that must be mastered before discharge"

A nurse is caring for a client who has SIADH. Which of the following findings should the nurse expect? Select all that apply. A. Decreased serum sodium B. Urine specific gravity of 1.001 C. Serum osmolarity 230 MOSM/liter D. Polyuria E. increased thirst

A, C A decrease in serum sodium is caused by an increase in the secretion of ADH. A decrease in serum osmolarity is caused by an increase in the secretion of ADH.

What are the uses of antithyroid drugs? SELECT ALL THAT APPLY a. emergency treatment of thyrotoxicosis b. to treat myxedema coma c. preparation for thyroid surgery d. Graves disease

A, C, D myxedema coma is treated with thyroid drugs via IV therapy. Its a coma brought on my hypOthyroidism

Which of the following is thyroid drugs used for? SELECT ALL THAT APPLY a. hypothyroidism b. hyperthyroidism c. Grave's disease d. Myxedema coma e. emergency tx of thyrotoxicosis

A, D Graves disease and emergency tx of thyrotoxicosis is treated with ANTIthyroid drugs

The nurse is reviewing the health record of a client who has syndrome of inappropriate antidiuretic hormone (SIADH). Which of the following laboratory findings should the nurse anticipate? Select all that apply. A. Low serum sodium B. High serum potassium C. Decreased urine osmolality D. High urine sodium E. Increased urine specific gravity

A, D, E SIADH results in water retention, causing a low serum sodium level. SIADH results in water retention, causing a high urine sodium level. SIADH results in water retention, causing an increase in urine specific gravity.

A nurse is providing teaching to a client who has a new diagnosis of DI. Which of the following statements by the client requires further teaching? A. I can drink up to 2 quarts of fluid a day. B. I should expect to urinate frequently at night. C. I may experience headaches. D. I may experience a dry mouth.

A. Excessive thirst is a manifestation of DI. Consumption of 4 to 30 L per day can be expected, and fluid intake should not be limited.

Which actions describe the beneficial effects produced by sulfonylurea oral hypoglycemics? (Select all that apply.) A. Stimulate insulin secretion from beta cells B. Increase hepatic glucose production C. Enhance action of insulin in various tissues D. Inhibit breakdown of insulin by liver

A. Stimulate insulin secretion from beta cells B. Enhance action of insulin in various tissues D. Inhibit breakdown of insulin by liver

Which long-acting insulin mimics natural, basal insulin with no peak action and a duration of 24 hours? A. insulin glargine (Lantus) B. insulin glulisine (Apidra) C. regular insulin (Humulin R) D. NPH insulin

A. insulin glargine (Lantus)

Which of the following would be the initial substance responsible for thyroid hormone regulation? a. iodine intake b. thyrotropin-releasing hormone c. thyroid-stimulating hormone d. Levothyroxine

B Initial substance for thyroid hormone release is thyrotropin-releasing hormone form the hypothalamus. It then stimulates TSH.

An external insulin pump is prescribed for a client with DM. The client asks the nurse about the functioning of the pump. The nurse bases the response on the information that the pump: a. Gives small continuous dose of regular insulin subcutaneously, and the client can self-administer a bolus with an additional dosage from the pump before each meal. b. Is timed to release programmed doses of regular or NPH insulin into the bloodstream at specific intervals. c. Is surgically attached to the pancreas and infuses regular insulin into the pancreas, which in turn releases the insulin into the bloodstream. d. Continuously infuses small amounts of NPH insulin into the bloodstream while regularly monitoring blood glucose levels"

ANSWER A. An insulin pump provides a small continuous dose of regular insulin subcutaneously throughout the day and night, and the client can self-administer a bolus with additional dosage from the pump before each meal as needed. Regular insulin is used in an insulin pump. An external pump is not attached surgically to the pancreas.

A patient with type 2 diabetes has sensory neuropathy of the feet and legs and peripheral vascular disease evidenced by decreased peripheral pulses and dependent rubor. The nurse teaches the patient that a. the feet should be soaked in warm water on a daily basis. b. flat-soled leather shoes are the best choice to protect the feet from injury. c. heating pads should always be set at a very low temperature. d. over-the-counter (OTC) callus remover may be used to remove callus and prevent pressure.

B Rationale: The patient is taught to avoid high heels and that leather shoes are preferred. The feet should be washed, but not soaked, in warm water daily. Heating pad use should be avoided. Commercial callus and corn removers should be avoided; the patient should see a specialist to treat these problems.

A client is taking Humulin NPH insulin daily every morning. The nurse instructs the client that the mostlikely time for a hypoglycemic reaction to occur is: A) 2-4 hours after administration B) 4-12 hours after administration C) 16-18 hours after administration D) 18-24 hours after administration

B: Rationale: Humulin is an intermediate acting insulin. The onset of action is 1.5 hours, it peaks in 4-12 hours, and its duration is 24 hours. Hypoglycemic reactions to insulin are most likely to occur during the peak time.

Insulin is secreted by which of the following types of cells? a) Melanocytes b) Beta cells c) Basal cells d) Neural cells

Beta cells Explanation: Insulin is secreted by the beta cells, in the islets of Langerhans of the pancreas. In diabetes, cells may stop responding to insulin, or the pancreas may decrease insulin secretion or stop insulin production completely. Melanocytes are what give the skin its pigment. Neural cells transmit impulses in the brain and spinal cord. Basal cells are a type of skin cell.

"The nurse is caring for a patient whose blood glucose level is 55mg/dL. What is the likely nursing response? A. Administer a glucagon injection B. Give a small meal C. Administer 10-15 g of a carbohydrate D. Give a small snack of high protein food

C The client has low hypoglycemia. This is generally treated with a small snack.

The thyroid gland is dependent on the hypothalamic-pituitary axis for regulation. Increasing the levels of thyroid hormone (by taking replacement thyroid hormone) would a. increase hypothalamic release of thyrotropin-releasing hormone (TRH) b. increase pituitary release of thyroid-stimulating hormone c. suppress hypothalamic release of TRH d. stimulate the thyroid gland to produce more T3 and T4

C suppress hypothalamic release of TRH

In educating a client with diabetes, what response would reveal need for further education? A. I should avoid tights B. I should take good care of my toe nails C. I should not go more than 3 days without washing my feet D. I should avoid going barefoot and should wear clean socks

C) I should not go more than 3 days w/o washing my feet"The recommended self-care routine is to wash feet on a daily basis without soaking and carefully cleaning.

The nurse is caring for a hospitalized three-year-old admitted with a history of syndrome of inappropriate antidiuretic hormone (SIADH). He has just received his breakfast tray. Which food should the nurse remove from his tray? 1. Oatmeal 2. Yogurt 3. Biscuit 4. Cantaloupe

Correct Answer: 4 Rationale 1: A child with SAIDH may have carbohydrates and fiber, such as in oatmeal. Rationale 2: A child with SIADH may have dairy products, such as yogurt. Rationale 3: A child with SIADH may have carbohydrates, such as in a biscuit. Rationale 4: A child with SIADH is on a fluid restriction. Cantaloupe contains significant fluid volume, so it would not be a good food for this child to consume.

Which of the following persons would most likely be diagnosed with diabetes mellitus? A 44-year-old.. A. Caucasian Woman B. Asian Woman C. African-American woman D. Hispanic Male

Correct answer: African-American woman Rationale: Age-specific prevalence of diagnosed diabetes mellitus (DM) is higher for African-Americans and Hispanics than for Caucasians. Among those younger than 75, black women had the highest incidence.

A frail elderly patient with a diagnosis of type 2 diabetes mellitus has been ill with pneumonia. The cliet's intake has been very poor, and she is admitted to the hospital for observation and management as needed. What is the most likely problem with this patient? A. Insulin resistance has developed. B. Diabetic ketoacidosis is occuring. C. Hypoglycemia unawareness is developing. D. Hyperglycemic hyperosmolar non-ketotic coma.

D.Illness, especially with the frail elderly patient whose appetite is poor, can result in dehydration and HHNC. Insulin resisitance is inidcated by a daily insulin requirement of 200 units or more. Diabetic ketoacidosis, an acute metabolic condition, usually is caused by absent or markedly decreased amounts of insulin.

Which of the following is an age-related change that may affect diabetes and its management? a) Increased thirst b) Hypotension c) Increased bowel motility d) Decreased renal function

Decreased renal function Explanation: Decreased renal function affects the management of diabetes. Other age-related changes that may affect diabetes and its management include hypertension, decreased bowel motility, and decreased thirst.

Which combination of adverse effects should a nurse monitor for when administering I.V. insulin to a client with diabetic ketoacidosis? a) Hypokalemia and hypoglycemia b) Hypernatremia and hypercalcemia c) Hyperkalemia and hyperglycemia d) Hypocalcemia and hyperkalemia

Hypokalemia and hypoglycemia Explanation: Blood glucose needs to be monitored in clients receiving I.V. insulin because of the risk of hyperglycemia or hypoglycemia. Hypoglycemia might occur if too much insulin is administered. Hypokalemia, not hyperkalemia, might occur because I.V. insulin forces potassium into cells, thereby lowering the plasma level of potassium. Calcium and sodium levels aren't affected by I.V. insulin administration.

Risk factors for type 2 diabetes include all of the following except: a. Advanced age b. Obesity c. Smoking d. Physical inactivity

Smoking Additional risk factors for type 2 diabetes are a family history of diabetes, impaired glucose metabolism, history of gestational diabetes, and race/ethnicity. African-Americans, Hispanics/Latinos, Asian Americans, Native Hawaiians, Pacific Islanders, and Native Americans are at greater risk of developing diabetes than whites.

Beta-blockers are used in the treatment of hyperthyroidism to counteract which of the following effects? a) Respiratory effects b) Parasympathetic c) Sympathetic d) Gastrointestinal effects

Sympathetic Explanation: Beta-adrenergic blocking agents are important in controlling the sympathetic nervous system effects of hyperthyroidism. For example, propranolol is used to control nervousness, tachycardia, tremor, anxiety, and heat intolerance.

The nurse is reviewing the actions that a patient with type 1 diabetes mellitus should take if mild hypoglycemia is experienced. What should the nurse include in this​ teaching? Select all that apply. a. Ingest additional 15 grams of carbohydrate if blood glucose remains low after 15 minutes. b. Ingest 4 ounces of fruit juice when blood glucose is below 70​ mg/dL. c. Measure blood glucose 15 minutes after ingesting a carbohydrate source. d. Add table sugar to 8 ounces of fruit juice when blood glucose is below 70​ mg/dL. e. Test blood glucose 30 minutes after reaching 70​ mg/dL.

a. Ingest additional 15 grams of carbohydrate if blood glucose remains low after 15 minutes. b. Ingest 4 ounces of fruit juice when blood glucose is below 70​ mg/dL. c. Measure blood glucose 15 minutes after ingesting a carbohydrate source.

A client is diagnosed with type 2 diabetes mellitus. What information about type 2 diabetes mellitus should the nurse include when providing client​ education? a. Insulin resistance occurs in peripheral tissues. b. The onset of hyperglycemia is rapid. c. Metabolism of dietary carbohydrates is enhanced. d. The liver suppresses glucose being released.

a. Insulin resistance occurs in peripheral tissues.

The nurse is caring for a client with diabetes mellitus who is at risk for developing chronic complications. For which conditions should the nurse​ monitor? Select all that apply. a. Nephropathy b. Stroke c. Dementia d. Retinopathy e. Neuropathy

a. Nephropathy b. Stroke d. Retinopathy e. Neuropathy

Which of the following symptoms would you ask about while taking a health history to help determine if a client has developed complications from diabetes​ mellitus? Select all that apply. a. Numbness in the feet b. Dizziness c. Frequent voiding d. Quick wound healing e. Vision changes

a. Numbness in the feet b. Dizziness c. Frequent voiding e. Vision changes

The nurse notes that a patient with type 2 diabetes mellitus is not prescribed aspirin 81 mg as recommended for the prevention of cardiovascular complications. What information in the​ patient's health history should the nurse use to understand why this medication has not been prescribed for the​ patient? Select all that apply. a. Patient prescribed warfarin​ (Coumadin) 5 mg by mouth every day. b. Patient receives a vitamin B12 injection every month. c. Patient develops a rash and urticaria when taking medications with sulfa. d. Patient treated for chronic alcoholism and liver cirrhosis the past year. e. Patient admitted for gastrointestinal bleeding 3 months ago.

a. Patient prescribed warfarin​ (Coumadin) 5 mg by mouth every day. d. Patient treated for chronic alcoholism and liver cirrhosis the past year. e. Patient admitted for gastrointestinal bleeding 3 months ago.

Which of the following are manifestations of type 1 diabetes​ mellitus? Select all that apply. a. Weight gain b. Glucosuria c. Blurred vision d. Fever e. Polyuria

b. Glucosuria c. Blurred vision e. Polyuria

The nurse is completing a health history on a client with type 1 diabetes mellitus. Which manifestations indicate the client is experiencing​ hyperglycemia? Select all that apply. a. Weight gain b. Polyphagia c. Polydipsia d. Fatigue e. Polyuria

b. Polyphagia c. Polydipsia d. Fatigue e. Polyuria

When caring for a patient with nephrogenic DI, the nurse would expect treatment to include a. fluid restriction b. thiazide diuretics c. a high-sodium diet d. chlorpropamide (DIabinese)

b. thiazide diuretics (Rationale- in nephrogenic Di the kidney is unable to respond to ADH, so vasopressin or hormone analogs are not effective. Thiazide diuretics slow the glomerular filtration rate in the kidney and produce a decrease in urine output. Low-sodium diets are also thought to decrease urine output. Fluids are not restricted, because the patient could become easily dehydrated.)

A patient with diabetes is​ diaphoretic, has a heart rate of 112 beats per​ minute, and is feeling nervous and shaky. What action should the nurse take​ first? a. Provide the patient with a snack of milk and crackers. b. Contact the laboratory and order a serum glucose level. c. Obtain a capillary serum glucose level reading with a glucose meter. d. Administer insulin utilizing the prescribed sliding scale dosages.

c. Obtain a capillary serum glucose level reading with a glucose meter.

In a patient with central diabetes insipidus, administration of aqueous vasopressin during a water deprivation test will result in a a. decrease in body weight b. increase in urinary output c. decrease in blood pressure d. increase in urine osmolality

d. increase in urine osmolality (rationale- a patient with DI has a deficiency of ADH with excessive loss of water from the kidney, hypovolemia, hypernatreamia, and dilute urine with a low specific gravity. When vasopressin is administered, the symptoms are reversed, with water retention, decreased urinary output that increases urine osmolality, and an increase in blood pressure.)

A patient with no previous history of diabetes mellitus has ketones in the urine. Which question should the nurse ask this​ patient? a. ​"Have you donated blood​ recently?" ​b. "Have you ever been told you have albumin in your​ urine?" ​c. "What did you eat for breakfast and lunch​ today?" d. ​"Can you please describe any weight loss strategies​ you've been​ using?"

d. ​"Can you please describe any weight loss strategies​ you've been​ using?"

A nurse is reviewing the health record of the client Who has HHS. Which of the following data confirms the diagnosis? Select all that apply. A. Evidence of recent myocardial infarction B. BUN 35 mg/dL C. Takes a calcium channel blocker D. Age of 77 years E. No insulin production

A, B, C, D

After the home health nurse has taught a patient and family about how to use glargine and regular insulin safely, which action by the patient indicates that the teaching has been successful? a. The patient disposes of the open insulin vials after 4 weeks. b. The patient draws up the regular insulin in the syringe and then draws up the glargine. c. The patient stores extra vials of both types of insulin in the freezer until needed. d. The patient's family prefills the syringes weekly and stores them in the refrigerator.

A Rationale: Insulin can be stored at room temperature for 4 weeks. Glargine should not be mixed with other insulins or prefilled and stored. Freezing alters the insulin molecule and should not be done.

A patient using a split mixed-dose insulin regimen asks the nurse about the use of intensive insulin therapy to achieve tighter glucose control. The nurse should teach the patient that a. intensive insulin therapy requires three or more injections a day in addition to an injection of a basal long-acting insulin. b. intensive insulin therapy is indicated only for newly diagnosed type 1 diabetics who have never experienced ketoacidosis. c. studies have shown that intensive insulin therapy is most effective in preventing the macrovascular complications characteristic of type 2 diabetes. d. an insulin pump provides the best glucose control and requires about the same amount of attention as intensive insulin therapy.

A Rationale: Patients using intensive insulin therapy must check their glucose level four to six times daily and administer insulin accordingly. A previous episode of ketoacidosis is not a contraindication for intensive insulin therapy. Intensive insulin therapy is not confined to type 2 diabetics and would prevent microvascular changes as well as macrovascular changes. Intensive insulin therapy and an insulin pump are comparable in glucose control.

Cardiac monitoring is initiated for a patient in diabetic ketoacidosis (DKA). The nurse recognizes that this measure is important to identify a. electrocardiographic (ECG) changes and dysrhythmias related to hypokalemia. b. fluid overload resulting from aggressive fluid replacement. c. the presence of hypovolemic shock related to osmotic diuresis. d. cardiovascular collapse resulting from the effects of hyperglycemia.

A Rationale: The hypokalemia associated with metabolic acidosis can lead to potentially fatal dysrhythmias such as ventricular tachycardia and ventricular fibrillation, which would be detected with ECG monitoring. Fluid overload, hypovolemia, and cardiovascular collapse are possible complications of DKA, but cardiac monitoring would not detect theses.

When teaching a patient with type 2 diabetes about taking glipizide (Glucotrol), the nurse determines that additional teaching about the medication is needed when the patient says, a. "Since I can take oral drugs rather than insulin, my diabetes is not serious and won't cause many complications." b. "If I overeat at a meal, I will still take just the usual dose of medication." c. "If I become ill, I may have to take insulin to control my blood sugar." d. "I should check with my doctor before taking any other medications because there are many that will affect glucose levels."

A Rationale: The patient should understand that type 2 diabetes places the patient at risk for many complications and that good glucose control is as important when taking oral agents as when using insulin. The other statements are accurate and indicate good understanding of the use of glipizide.

A nurse in an intensive care unit is admitting a client who has myxedema coma. Which of the following should the nurse anticipate in caring for this client? Select all that apply. A. Observe cardiac monitor for inverted T-wave B. Observe evidence of urinary tract infection C. Initiate IV fluids using 0.9% sodium chloride D. Expect a prescription for levothyroxine IV bolus E. Provide warmth using a heating pad

A, B, C, D The client who has myxedema you may have a flat or inverted T-wave, as well as ST deviations; an infection such as a UTI, may precipitate myxedema coma; hyponatremia is a typical finding the presence of myxedema coma. Therefore, intravenous therapy is administered using either isotonic hypertonic fluids. Myxedema coma is a severe complication of hypothyroidism and can lead to come or death. Levothyroxine is administered IV bolus to treat the condition.

Hypothyroidism is a very common and often missed disorder. Signs and symptoms of hypothyroidism include: Select All That Apply a. increased body temperature b. thickening of the tongue c. bradycardia d. loss of hair e. excessive weight loss f. oily skin

A, B, D incr body temp thickening tongue loss of hair

The nurse is preparing to receive a client from the PACU who is post operative following a thyroidectomy. The nurse should ensure that which of the following equipment is available? Select all that apply. A. Suction equipment B. Humidified air C. Flashlight D. Tracheostomy tray E. 02 delivery equipment

A, B, D, E

A patient with newly diagnosed type 2 diabetes mellitus asks the nurse what ""type 2"" means in relation to diabetes. The nurse explains to the patient that type 2 diabetes differs from type 1 diabetes primarily in that with type 2 diabetes a. the pt is totally dependent on an outside source of insulin b. there is a decreased insulin secretion and cellular resistance to insulin that is produced c. the immune system destroys the pancreatic insulin-producing cells d. the insulin precurosr that is secreted by the pancreas is not activated by the liver

Answer B - Rationale: In type 2 diabetes, the pancreas produces insulin, but the insulin is insufficient for the body's needs or the cells do not respond to the insulin appropriately. The other information describes the physiology of type 1 diabetes

A patient receiving propylthiouracil (PTU) asks the nurse how this medication will help relieve his symptoms. What is the nurse's best response? A. "Propylthiouracil inactivates any circulating thyroid hormone, thus decreasing signs and symptoms of hyperthyroidism." B. "Propylthiouracil inhibits the formation of new thyroid hormone, thus gradually returning your metabolism to normal." C. "Propylthiouracil helps your thyroid gland use iodine and synthesize hormones better." D. "Propylthiouracil stimulates the pituitary gland to secrete thyroid-stimulating hormone (TSH), which inhibits the production of hormones by the thyroid gland."

B. "Propylthiouracil inhibits the formation of new thyroid hormone, thus gradually returning your metabolism to normal."

The guidelines for Carbohydrate Counting as medical nutrition therapy for diabetes mellitus includes all of the following EXCEPT: a. Flexibility in types and amounts of foods consumed b. Unlimited intake of total fat, saturated fat and cholesterol c. Including adequate servings of fruits, vegetables and the dairy group d. Applicable to with either Type 1 or Type 2 diabetes mellitusb. Unlimited intake of total fat, saturated fat and cholesterol

B. You want to be careful of how much you eat in any food group.

When caring for a patient newly diagnosed with gestational diabetes, the nurse would question an order for which drug? A. insulin glargine (Lantus) B. glipizide (Glucotrol) C. insulin glulisine (Apidra) D. NPH insulin

B. glipizide (Glucotrol)

The nurse will advise the patient to treat hypoglycemia with which drug? A. propranolol (Inderal) B. glucagon C. acarbose (Precose) D. bumetanide (Bumex)

B. glucagon

A diagnosis of hyperglycemic hyperosmolar nonketotic coma (HHNC) is made for a patient with type 2 diabetes who is brought to the emergency department in an unresponsive state. The nurse will anticipate the need to a. administer glargine (Lantus) insulin. b. initiate oxygen by nasal cannula. c. insert a large-bore IV catheter. d. give 50% dextrose as a bolus.

C Rationale: HHNC is initially treated with large volumes of IV fluids to correct hypovolemia. Regular insulin is administered, not a long-acting insulin. There is no indication that the patient requires oxygen. Dextrose solutions will increase the patient's blood glucose and would be contraindicated.

A diabetic patient is admitted with ketoacidosis and the health care provider writes all of the following orders. Which order should the nurse implement first? a. Start an infusion of regular insulin at 50 U/hr. b. Give sodium bicarbonate 50 mEq IV push. c. Infuse 1 liter of normal saline per hour. d. Administer regular IV insulin 30 U.

C Rationale: The most urgent patient problem is the hypovolemia associated with DKA, and the priority is to infuse IV fluids. The other actions can be accomplished after the infusion of normal saline is initiated.

A client is brought to the emergency department in an unresponsive state, and a diagnosis of hyperglycemic hyperosmolar nonketotic syndrome is made. The nurse would immediately prepare to initiate which of the following anticipated physician's prescriptions? 1. Endotracheal intubation 2. 100 units of NPH insulin 3. Intravenous infusion of normal saline 4. Intravenous infusion of sodium bicarbonate

CORRECT ANSWER: 3. Intravenous infusion of normal saline Rationale: The primary goal of treatment is hyperglycemic hyperosmolar nonketotic syndrome (HHNS) is to rehydrate the client to restore the fluid volume and to correct electrolyte deficiency. Intravenous fluid replacement is similar to that administered in diabetic keto acidosis (DKA) and begins with IV infusion of normal saline. Regular insulin, not NPH insulin, would be administered. The use of sodium bicarbonate to correct acidosis is avoided because it can precipitate a further drop in serum potassium levels. Intubation and mechanical ventilation are not required to treat HHNS.

The primary function of the thyroid gland includes which of the following? a) Control of cellular metabolic activity b) Reabsorption of water c) Reduction of plasma level of calcium d) Facilitation of milk ejection

Control of cellular metabolic activity Explanation: The primary function of the thyroid hormone is to control cellular metabolic activity. Oxytocin facilitates milk ejection during lactation and increases the force of uterine contraction during labor and delivery. Antidiuretic hormone (ADH) release results in reabsorption of water into the bloodstream rather than excretion by the kidneys. Calcitonin reduces the plasma level of calcium by increasing its deposition in bone.

Which of the following factors are risks for the development of diabetes mellitus? (Select all that apply.) a) Age over 45 years b) Overweight with a waist/hip ratio >1 c) Having a consistent HDL level above 40 mg/dl d) Maintaining a sedentary lifestyle

Correct: a,b,d"Rationale: Aging results in reduced ability of beta cells to respond with insulin effectively. Overweight with waist/hip ratio increase is part of the metabolic syndrome of DM II. There is an increase in atherosclerosis with DM due to the metabolic syndrome and sedentary lifestyle.

The nurse is teaching a patient with diabetes about​ self-management. What should the nurse include regarding medications to treat diabetes​ mellitus? a. Patients with type 1 diabetes may progress to type 2 if blood glucose levels are not well controlled. b. Patients with type 2 diabetes may achieve normal blood glucose levels with a combination of oral medications and insulin. c. Patients with type 2 diabetes will always need an exogenous source of insulin. d. Patients with type 1 diabetes may achieve normal blood glucose levels with oral medications.

b. Patients with type 2 diabetes may achieve normal blood glucose levels with a combination of oral medications and insulin.

A client with diabetes mellitus is admitted to the medical unit for chronic complications. The nurse ensures that the floor is free of​ clutter, uses a night​ light, and checks the water temperature before bathing the client. What potential problem do these interventions address when caring for this​ client? a. Ineffective coping b. Risk for injury c. Risk for infection d. Acute pain

b. Risk for injury

The nurse is reviewing data collected from a patient with a predisposition to developing insulin resistance. Which medications should the nurse identify as potentially causing this patient to develop​ diabetes? Select all that apply. a. furosemide​ (Lasix) b. nicotinic acid​ (Niacor) c. phenytoin​ (Dilantin) d. acetaminophen​ (Tylenol) e. levothyroxine​ (Synthroid)

a. furosemide​ (Lasix) b. nicotinic acid​ (Niacor) c. phenytoin​ (Dilantin) e. levothyroxine​ (Synthroid)

The nurse is caring for a healthy patient who has a serum glucose level of 60​ mg/dL. The nurse anticipates which counterregulatory serum hormonal changes to occur in this​ patient? Select all that apply. a. increased growth hormone levels b. decreased thyroxine levels c. decreased glucocorticoid levels d. increased epinephrine levels e. increased insulin levels

a. increased growth hormone levels d. increased epinephrine levels

The nurse is teaching a patient with type 2 diabetes mellitus about glyburide​ (DiaBeta). The nurse knows teaching has been effective when the patient​ states, "I need to monitor for​ dizziness, lightheadedness, and sweating if I​ take: Select all that apply. a. metoprolol​ (Lopressor) for​ hypertension." b. docusate sodium​ (Colace) for​ constipation." c. cetirizine​ (Zyrtec) for​ allergies." d. ranitidine​ (Zantac) for​ heartburn." e. ibuprofen​ (Motrin) for​ pain."

a. metoprolol​ (Lopressor) for​ hypertension." d. ranitidine​ (Zantac) for​ heartburn." e. ibuprofen​ (Motrin) for​ pain."

A client who is started on metformin and glyburide would have initially presented with which symptoms? "a. Polydispisa, polyuria, and weight loss b. weight gain, tiredness, & bradycardia c. irritability, diaphoresis, and tachycardia d. diarrhea, abdominal pain, and weight loss

a. Polydispisa, polyuria, and weight loss"Symptoms of hyperglycemia include polydipsia, polyuria, and weight loss. Metformin and sulfonylureas are commonly ordered medications. Weight gain, tiredness, and bradycardia are symptoms of hypothyroidism. Irritability, diaphoresis, and tachycardia are symptoms of hypoglycemia. Symptoms of Crohn's disease include diarrhea, abdominal pain, and weight loss.

A patient with diabetes mellitus has​ albuminuria, hypertension, and edema. What should the nurse expect to be prescribed for this​ patient? a. Review weight loss strategies. b. Increase salt intake. c. Provide antibiotic therapy as prescribed. d. Restrict activity.

a. Review weight loss strategies.

A client is admitted to the medical unit for possible diabetes mellitus. Which manifestations should the nurse monitor for that indicate the client is experiencing type 2 diabetes​ mellitus? Select all that apply. a. Skin infection b. Blurred vision c. Ketoacidosis d. Polydipsia e. Polyuria

a. Skin infection b. Blurred vision d. Polydipsia e. Polyuria

A​ 78-year-old patient without​ polyuria, polydipsia, or polyphagia has a serum glucose level of 130​ mg/dL. What should the nurse conclude about this​ patient? a. The patient will need to be assessed for other manifestations of diabetes. b. The patient has type 1 diabetes mellitus. c. The patient might have eaten a meal with high sugar content prior to the testing. d. The laboratory results might be erroneous.

a. The patient will need to be assessed for other manifestations of diabetes.

Gladys Lewis is a​ 48-year-old woman with a history of hypertension who is admitted to the hospital with an infected wound on her right ankle that does not seem to be healing. She reports that she has been experiencing​ fatigue, blurred​ vision, polyuria, and polydipsia for the past month. Upon reviewing her medical​ record, you note that she has a BMI of 32 and a blood glucose level of 225​ mg/dL. Her blood and urine are negative for ketones. Which of the following do you suspect may be happening to Ms.​ Lewis? a. Type 1 diabetes mellitus b. Type 2 diabetes mellitus c. Diabetes insipidus d. Hypoglycemia

b. Type 2 diabetes mellitus

A nurse is preparing to administer IV fluids to client who has DKA. Which of the following is an appropriate nursing action? A. Administering IV infusion of regular insulin at 0.3 units per kilogram per hour B. Administer and IV infusion of 0.45% sodium chloride C. Rapidly administering IV infusion of 0.9% sodium chloride D. Add glucose to the IV infusion when serum glucose is 350 mg/dL

c

When planning care for a client with diabetes​ mellitus, the nurse addresses the potential problem of risk for infection. Which intervention will assist in addressing this potential​ problem? a. Monitor sensation to extremities daily. b. Instruct the client to have an oral examination yearly. c. Teach using lukewarm water and soap for skin care. d. Promote smoking cessation.

c. Teach using lukewarm water and soap for skin care.

Which of the following may be a manifestation of diabetes mellitus in the older​ client? a. Palpitations b. Increased thirst c. Urinary incontinence d. Increased hunger

c. Urinary incontinence

The nurse is providing education to a client recently diagnosed with type 1 diabetes mellitus. What information will the nurse include when teaching about the characteristics of this​ disease? a. Hyperosmolar hyperglycemia state b. Insulin resistance c. Rapid onset of disease d. Breakdown of fats and proteins

d. Breakdown of fats and proteins

In a patient with central diabetes insipidus, administration of aqueous vasopressin during a water deprivation test will result in a a. decrease in body weight b. increase in urinary output c. decrease in blood pressure d. increase in urine osmolality

d. increase in urine osmolality (rationale- a patient with DI has a deficiency of ADH with excessive loss of water from the kidney, hypovolemia, hypernatreamia, and dilute urine with a low specific gravity. When vasopressin is administered, the symptoms are reversed, with water retention, decreased urinary output that increases urine osmolality, and an increase in blood pressure.)

A patient beginning insulin for type 2 diabetes is experiencing blurred vision and is concerned about becoming blind. What response by the nurse is most​ appropriate? a. ​"Blurry vision is very common. Do not​ worry." b. ​"I will call the physician to report your​ symptoms." c. ​"I will make an appointment for you to see an​ ophthalmologist." d. ​"This is a normal response when insulin therapy is​ initiated."

d. ​"This is a normal response when insulin therapy is​ initiated."

The nurse is caring for a patient with type 1 diabetes mellitus. Which patient statement requires immediate intervention by the​ nurse? a. ​"I will not use insulin detemir in my insulin​ pump." b. ​"I won't mix my cloudy regular insulin with other​ insulins." ​c. "I will take my lispro insulin 15 minutes before I eat​ breakfast." d. ​"I am allergic to​ eggs."

b. ​"I won't mix my cloudy regular insulin with other​ insulins."

The nurse is educating a client newly diagnosed with type 1 diabetes mellitus. What information should the nurse include in client education about ongoing monitoring of glucose​ levels? a. Urine testing will assist in measuring hypoglycemia. b. Urine testing is only used until glucose goals are achieved. ​c. Self-monitoring blood glucose is painless and noninvasive. d. ​Self-monitoring blood glucose should occur three or four times a day.

d. ​Self-monitoring blood glucose should occur three or four times a day.

The nurse is reviewing laboratory reports of a client who has HHS. Which of the following is an expected finding? A. Serum pH 7.2 B. Serum osmolarity 350 mOsm/L C. Serum potassium 3.8 mg/dL D. Serum creatinine 0.8 mg/dL

B

The nurse administered 28 units of Humulin N, an intermediate-acting insulin, to a client diagnosed with Type 1 diabetes at 1600. Which action should the nurse implement? 1. Ensure the client eats the bedtime snack. 2. Determine how much food the client ate at lunch. 3. Perform a glucometer reading at 0700. 4. Offer the client protein after administering insulin.

1: ensure the client eats the bedtime snack" 1. Humulin N peaks in 6-8 hours, making the client at risk for hypoglycemia around midnight, which is why the client should receive a bedtime snack. This snack will prevent nighttime hypoglycemia. (Correct) 2. The food intake at lunch will not affect the client's blood glucose level at midnight. 3. The client's glucometer reading should be done around 2100 to assess the effectiveness of insulin at 1600. 4. Humulin N is an intermediate-acting insulin that has an onset in 2-4 hours but does not peak until 6-8 hours.

The nurse is discussing the importance of exercising to a client diagnosed with Type 2diabetes whose diabetes is well controlled with diet and exercise. Which informationshould the nurse include in the teaching about diabetes? 1.Eat a simple carbohydrate snack before exercising. 2.Carry peanut butter crackers when exercising. 3.Encourage the client to walk 20 minutes three (3) times a week. 4.Perform warmup and cooldown exercises

4.Perform warmup and cooldown exercises "The client diagnosed with Type 2 diabetes whois not taking insulin or oral agents does notneed extra food before exercise.2.The client with diabetes who is at risk forhypoglycemia when exercising should carry asimple carbohydrate, but this client is not atrisk for hypoglycemia.3.Clients with diabetes that is controlled by dietand exercise must exercise daily at the sametime and in the same amount to control theglucose level. 4. [correct] All clients who exercise should perform warmup and cooldown exercises to helpprevent muscle strain and injury"

Intramuscular glucagon is administered to an unresponsive patient for treatment of hypoglycemia. Which action should the nurse take after the patient regains consciousness? a. Give the patient a snack of cheese and crackers. b. Have the patient drink a glass of orange juice or nonfat milk. c. Administer a continuous infusion of 5% dextrose for 24 hours. d. Assess the patient for symptoms of hyperglycemia.

A Rationale: Rebound hypoglycemia can occur after glucagon administration, but having a meal containing complex carbohydrates plus protein and fat will help prevent hypoglycemia. Orange juice and nonfat milk will elevate blood sugar rapidly, but the cheese and crackers will stabilize blood sugar. Administration of glucose intravenously might be used in patients who were unable to take in nutrition orally. The patient should be assessed for symptoms of hypoglycemia after glucagon administration.

While hospitalized and recovering from an episode of diabetic ketoacidosis, the patient calls the nurse and reports feeling anxious, nervous, and sweaty. Based on the patient's report, the nurse should a. obtain a glucose reading using a finger stick. b. administer 1 mg glucagon subcutaneously. c. have the patient eat a candy bar. d. have the patient drink 4 ounces of orange juice.

A Rationale: The patient's clinical manifestations are consistent with hypoglycemia and the initial action should be to check the patient's glucose with a finger stick or order a stat blood glucose. If the glucose is low, the patient should ingest a rapid-acting carbohydrate, such as orange juice. Glucagon might be given if the patient's symptoms become worse or if the patient is unconscious. Candy bars contain fat, which would slow down the absorption of sugar and delay the response to treatment.

Amitriptyline (Elavil) is prescribed for a diabetic patient with peripheral neuropathy who has burning foot pain occurring mostly at night. Which information should the nurse include when teaching the patient about the new medication? a. Amitriptyline will help prevent the transmission of pain impulses to the brain. b. Amitriptyline will improve sleep and make you less aware of nighttime pain. c. Amitriptyline will decrease the depression caused by the pain. d. Amitriptyline will correct some of the blood vessel changes that cause pain.

A Rationale: Tricyclic antidepressants decrease the transmission of pain impulses to the spinal cord and brain. Tricyclics also improve sleep quality and are used for depression, but that is not the major purpose for their use in diabetic neuropathy. The blood vessel changes that contribute to neuropathy are not affected by tricyclics.

When assessing the patient experiencing the onset of symptoms of type 1 diabetes, which question should the nurse ask? a. "Have you lost any weight lately?" b. "Do you crave fluids containing sugar?" c. "How long have you felt anorexic?" d. "Is your urine unusually dark-colored?"

A) lost any weight? a. Weight loss occurs because the body is no longer able to absorb glucose and starts to break down protein and fat for energy. b. The patient is thirsty but does not necessarily crave sugar- containing fluids. c. Increased appetite is a classic symptom of type 1 diabetes. d. With the classic symptom of polyuria, urine will be very dilute.

A client with type 1 diabetes presents with a decreased level of consciousness and a fingerstick glucose level of 39 mg/dl. His family reports that he has been skipping meals in an effort to lose weight. Which nursing intervention is most appropriate? a) Observing the client for 1 hour, then rechecking the fingerstick glucose level b) Inserting a feeding tube and providing tube feedings c) Administering a 500-ml bolus of normal saline solution d) Administering 1 ampule of 50% dextrose solution, per physician's order

Administering 1 ampule of 50% dextrose solution, per physician's order Explanation: The nurse should administer 50% dextrose solution to restore the client's physiological integrity. Feeding through a feeding tube isn't appropriate for this client. A bolus of normal saline solution doesn't provide the client with the much-needed glucose. Observing the client for 1 hour delays treatment. The client's blood glucose level could drop further during this time, placing him at risk for irreversible brain damage.

Administration of propylthiouracil would include giving the drug a. once a day in the morning b. around the clock to assure therapeutic levels c. once a day at bedtime to decrease adverse effects d. if the patient is experiencing slow heart rate, skin rash, or excessive bleeding

B Around the clock to assure therapeutic levels

A newly diagnosed type 1 diabetic patient likes to run 3 miles several mornings a week. Which teaching will the nurse implement about exercise for this patient? a. "You should not take the morning NPH insulin before you run." b. "Plan to eat breakfast about an hour before your run." c. "Afternoon running is less likely to cause hypoglycemia." d. "You may want to run a little farther if your glucose is very high."

B Rationale: Blood sugar increases after meals, so this will be the best time to exercise. NPH insulin will not peak until mid-afternoon and is safe to take before a morning run. Running can be done in either the morning or afternoon. If the glucose is very elevated, the patient should postpone the run.

A 1200-calorie diet and exercise are prescribed for a patient with newly diagnosed type 2 diabetes. The patient tells the nurse, "I hate to exercise! Can't I just follow the diet to keep my glucose under control?" The nurse teaches the patient that the major purpose of exercise for diabetics is to a. increase energy and sense of well-being, which will help with body image. b. facilitate weight loss, which will decrease peripheral insulin resistance. c. improve cardiovascular endurance, which is important for diabetics. d. set a successful pattern, which will help in making other needed changes.

B Rationale: Exercise is essential to decrease insulin resistance and improve blood glucose control. Increased energy, improved cardiovascular endurance, and setting a pattern of success are secondary benefits of exercise, but they are not the major reason.

A nurse is providing instructions to client who has Graves' disease and has a new prescription for propanolol (Inderal). Which of the following information should the nurse include? A. An adverse affects of this medication is jaundice. B. Take your pulse before each dose. C. The purpose of this medication is to decrease production of thyroid hormones. D. You should stop taking the medication if you have a sore throat.

B. Propanolol can cause bradycardia. The client should take his pulse before each dose. If there is a significant change, he should withhold the dose and consult his provider.

A nurse is caring for a client who has DI. Which of the following urinalysis laboratory findings should the nurse anticipate? A. Absence of glucose B. Decreased specific gravity C. Presence of ketones D. Presence of RBCs

B. The urine of a client who has DI will be dilute with a urine specific gravity of less than 1.005.

Which of the following would the nurse expect to assess in a patient experiencing hyperthyroidism? a. slow and deep tendon reflexes b. bradycardia c. flushed, warm skin d. intolerance to cold

C flushed warm skin

A diabetic patient has a serum glucose level of 824 mg/dL (45.7 mmol/L) and is unresponsive. Following assessment of the patient, the nurse suspects diabetic ketoacidosis rather than hyperosmolar hyperglycemic syndrome based on the finding of a. polyuria b. severe dehydration c. rapid, deep respirations d. decreased serum potassium

C is correct, Signs and symptoms of DKA include manifestations of dehydration such as poor skin turgor, dry mucous membranes, tachycardia, and orthostatic hypotension. Early symptoms may include lethargy and weakness. As the patient becomes severely dehydrated, the skin becomes dry and loose, and the eyeballs become soft and sunken. Abdominal pain is another symptom of DKA that may be accompanied by anorexia and vomiting. Kussmaul respirations (i.e., rapid, deep breathing associated with dyspnea) are the body's attempt to reverse metabolic acidosis through the exhalation of excess carbon dioxide. Acetone is identified on the breath as a sweet, fruity odor. Laboratory findings include a blood glucose level greater than 250 mg/dL, arterial blood pH less than 7.30, serum bicarbonate level less than 15 mEq/L, and moderate to large ketone levels in the urine or blood ketones.

A nurse is assessing a client who is 12 hr postoperative following a thyroidectomy. Which of the following findings are indicative of a thyroid crisis? Select all that apply. A. Bradycardia B. Hypothermia C. Tremors D. Abdominal pain E. Mental confusion

C, D, E Excessive levels of thyroid hormone can cause a client to experience tremors. When thyroid crisis occurs, the client can experience G.I. conditions, such as vomiting, diarrhea, and abd pain. Excessive thyroid hormone levels can cause the client to experience mental confusion.

The nurse is teaching care to client who has DM. Which of the following information should the nurse include in the teaching? Select all that apply. A. Remove calluses using over-the-counter remedies B. Apply lotion between toes C. Perform nail care after bathing D. Trim toenails straight across E. Wear closed-toe shoes

C, D, E perform nail care after bathing, when toenails are softer and easier to trim Trim toenails straight across to prevent injury to soft tissues of the toes. Wear closed-toe shoes to prevent injury to soft tissue the toes and feet.

The nurse is teaching a teenage client newly diagnosed with type 1diabetes about complications of the disease. The nurse explains that clients with type 1 diabetes can avoid lipoatrophy by: 1. Rotating injection sites. 2. Checking blood sugars at mealtime and bedtime. 3. Using a sliding scale for additional coverage. 4. Administration of insulin via insulin pump.

Correct Answer: 1 Rationale 1: Lipoatrophy is caused by using the same insulin injection site. Rationale 2: Checking blood sugars does not influence lipoatrophy. Rationale 3: A sliding scale does not influence lipoatrophy. Rationale 4: Insulin administration via pump doesn't influence lipoatrophy.

A child weighing 18.2 kg with a history of diabetes insipidus has been admitted to the hospital. Which of the physician's orders would the nurse question? 1. Stat electrolytes 2. Urine specific gravity with each void 3. DDAVP (desmopressin) PO 4. Restrict oral fluids to 500 mL every 24 hours.

Correct Answer: 4 Rationale 1: Stat electrolytes would be an appropriate order to check for hypernatremia. Rationale 2: Urine specific gravity is checked because it is often low. Rationale 3: DDAVP is the drug of choice for a child with DI. Rationale 4: Fluid replacement, not fluid restriction, is necessary for child with DI.

A middle-aged female client complains of anxiety, insomnia, weight loss, the inability to concentrate, and eyes feeling "gritty." Thyroid function tests reveal the following: thyroid-stimulating hormone (TSH) 0.02 U/ml, thyroxine 20 g/dl, and triiodothyronine 253 ng/dl. A 6-hour radioactive iodine uptake test showed a diffuse uptake of 85%. Based on these assessment findings, the nurse should suspect: a) Hashimoto's thyroiditis. b) multinodular goiter. c) Graves' disease. d) thyroiditis.

Graves' disease. Explanation: Graves' disease, an autoimmune disease causing hyperthyroidism, is most prevalent in middle-aged females. In Hashimoto's thyroiditis, the most common form of hypothyroidism, TSH levels would be high and thyroid hormone levels low. In thyroiditis, radioactive iodine uptake is low (?2%), and a client with a multinodular goiter will show an uptake in the high-normal range (3% to 10%).

Which electrolyte replacement should the nurse anticipate being ordered by thehealth-care provider in the client diagnosed with DKA who has just been admitted to the ICD? 1.Glucose. 2.)Potassium. 3.Calcium. 4.Sodium

Potassium"1.Glucose is elevated in DKA; therefore, theHCP would not be replacing glucose. 2.(CORRECT)-->The client in DKA loses potassium from increased urinary output, acidosis, cata-bolic state, and vomiting. Replacement isessential for preventing cardiac dysrhyth-mias secondary to hypokalemia. 3.Calcium is not affected in the client with DKA.4.The IV that is prescribed 0.9% normal salinehas sodium, but it is not specifically orderedfor sodium replacement. This is an isotonicsolution. TEST-TAKING HINT: Option "1" should be elim-inated because the problem with DKA iselevated glucose so the HCP would not bereplacing it. The test taker should use physiol-ogy knowledge and realize potassium is in thecell."

The nurse is assessing a patient who has a family history of type 2 diabetes mellitus. Which finding would require​ follow-up by the​ nurse? a. decreased​ waist-to-hip ratio through dietary changes b. a new prescription for levothyroxine​ (Synthroid) for hypothyroidism c. a fasting blood glucose level of 89​ mg/dL d. delivery of a baby that weighed 8 pounds and 12 ounces

b. a new prescription for levothyroxine​ (Synthroid) for hypothyroidism

During care of a patient with syndrome of inappropriate ADH (SIADH), the nurse should a. monitor neurologic status Q2H or more often if needed b. keep the head of the bed elevated to prevent ADH release c. teach the patient receiving treatment with diuretics to restrict sodium intake d. notify the physician if the patient's blood pressure decreases more than 20mmHg from baseline

a. monitor neurologic status Q2H or more often if needed Rationale- the patient with SIADH has marked dilution hyponatremia and should be monitored for decreased neurologic function and convulsions every 2 hours. ADH release is reduced by keeping the head of the bed flat to increase left atrial filling pressure, and sodium intake is supplemented because of hyponatremia and sodium loss caused by diuretics. A reduction in blood pressure indicates a reduction in total fluid volume and is an expected outcome of treatment.)

Which are symptoms of hypoglycemia? A. irritability B. drowsiness c. Abdominal pain D. nausea and vomiting

A. Irritability: signs of hypoglycemia include irritability, shaky feeling, hunger, headache, dizziness. Other symptoms are hyperglycemia.

A nurse in a providers office is reviewing the health record of a client who is being evaluated for Graves' disease. Which of the following is an expected laboratory finding for this client? A. Decreased thyrotropin receptor antibodies B. Decreased thyroid stimulating hormone C. Decreased free thyroxine index D. Decreased triiodothyronine

B. In the presence of Graves' disease, a low thyroid stimulating hormone (TSH) is an expected finding. The pituitary gland increases the production of TSH when thyroid hormone levels are elevated.

During a clinic visit 3 months following a diagnosis of type 2 diabetes, the patient reports following a reduced-calorie diet. The patient has not lost any weight and did not bring the glucose-monitoring record. The nurse will plan to obtain a(n) a. fasting blood glucose level. b. urine dipstick for glucose. c. glycosylated hemoglobin level. d. oral glucose tolerance test.

C Rationale: The glycosylated hemoglobin (Hb A1C) test shows the overall control of glucose over 90 to 120 days. A fasting blood level indicates only the glucose level at one time. Urine glucose testing is not an accurate reflection of blood glucose level and does not reflect the glucose over a prolonged time. Oral glucose tolerance testing is done to diagnose diabetes, but is not used for monitoring glucose control once diabetes has been diagnosed.

Goiter, or enlargement of the thyroid gland, is usually associated with a. hypothyroidism b. iodine deficiency c. hyperthyroidism d. underactive thyroid tissue

C hyperthyroidism

The nurse teaches the diabetic patient who rides a bicycle to work every day to administer morning insulin into the a. thigh. b. buttock. c. arm. d. abdomen.

D Rationale: Patients should be taught not to administer insulin into a site that will be exercised because exercise will increase the rate of absorption. The thigh, buttock, and arm are all exercised by riding a bicycle.

A nurse explains to a client with thyroid disease that the thyroid gland normally produces: a) TSH, triiodothyronine (T3), and calcitonin. b) thyrotropin-releasing hormone (TRH) and TSH. c) T3, thyroxine (T4), and calcitonin. d) iodine and thyroid-stimulating hormone (TSH).

T3, thyroxine (T4), and calcitonin. Explanation: The thyroid gland normally produces thyroid hormone (T3 and T4) and calcitonin. The pituitary gland produces TSH to regulate the thyroid gland. The hypothalamus gland produces TRH to regulate the pituitary gland.

The nurse instructs a patient with type 2 diabetes mellitus on the use of a glucometer for​ self-monitoring. Which patient statements about glucometer performance indicate that teaching has been​ effective? Select all that apply. a. Grapefruit juice should not be ingested when using the glucometer. b. A patient with sickle cell anemia may need another way to check blood glucose levels. c. A sufficient amount of blood must be applied to the strip. d. Follow​ manufacturer's recommendation regarding cleaning of meter. e. Correctly apply the blood to the meter strip.

b. A patient with sickle cell anemia may need another way to check blood glucose levels. c. A sufficient amount of blood must be applied to the strip. d. Follow​ manufacturer's recommendation regarding cleaning of meter. e. Correctly apply the blood to the meter strip.

The nurse caring for a client with diabetes mellitus understands that this client is at risk for a number of complications. What complication is this client at greatest risk for​ developing? a. Bowel obstruction b. Coronary artery disease c. Acute renal disease d. Memory impairment

b. Coronary artery disease

The nurse is caring for a woman at 37 weeks gestation. The client was diagnosed with insulin-dependent diabetes mellitis (IDDM) at age 7. The client states, "I am so thrilled that I will be breastfeeding my baby." Which of the following responses by the nurse is best? 1. You will probably need less insulin while you are breastfeeding. 2. You will need to initially increase your insulin after the baby is born. 3. You will be able to take an oral hypoglycemic instead of insulin after the baby is born. 4. You will probably require the same dose of insulin that you are now taking."

"1. breastfeeding has an antidiabetogenic effect, less insulin is needed. (correct) 2. insulin needs will decrease due to antidiabetogenic effect of breastfeeding and physiological changes during immediate postpartum period. 3. client has IDDM, insulin required. 4. during third trimester insulin requirements increase due to increased insulin resistance"

Which of the following describe the therapeutic actions of antithyroid drugs? SELECT ALL THAT APPLY a. blocks synthesis of thyroid b. blocks conversion of T4 to T3 c. prevents oxidation of iodine d. blocks conversion of T3 to T4 e. initiates oxidation of iodine

A, B, C

Glyburide (Micronase, DiaBeta, Glynase) is prescribed for a patient whose type 2 diabetes has not been controlled with diet and exercise. When teaching the patient about glyburide, the nurse explains that a. glyburide stimulates insulin production and release from the pancreas. b. the patient should not take glyburide for 48 hours after receiving IV contrast media. c. glyburide should be taken even when the blood glucose level is low in the morning. d. glyburide decreases glucagon secretion.

A Rationale: The sulfonylureas stimulate the production and release of insulin from the pancreas. If the glucose level is low, the patient should contact the health care provider before taking the glyburide, since hypoglycemia can occur with this category of medication. Metformin should be held for 48 hours after administration of IV contract, but this is not necessary for glyburide.

A patient with DI is treated with nasal desmopression. The nurse recognize that the drug is not having an adequate therapeutic effect the the patient experiences a. headache and weight gain b. nasal irritation and nausea c. a urine specific gravity of 1.002 d. an oral intake greater than urinary output

A patient with DI is treated with nasal desmopression. The nurse recognize that the drug is not having an adequate therapeutic effect the the patient experiences a. headache and weight gain b. nasal irritation and nausea c. a urine specific gravity of 1.002 d. an oral intake greater than urinary output c. a urine specific gravity of 1.002 (rationale- normal urine specific gravity is 1.003 to 1.030, and urine with a specific gravity of 1.002 is very dilute, indicating that there continues to be excessive loss of water and that treatment of DI is inadequate. H/A, weight gain, and oral intake greater the urinary output are signs of volume excess that occur with overmedication. Nasal irritation & nausea may also indicate overmedication.)

Side effects of thyroid drugs include: SELECT ALL THAT APPLY a. anxiety b. hypertension c. narcolepsy d. skin rash e. metallic taste in mouth

A, B thyroid drugs cause sleeplessness not narcolepsy skin rash and metallic taste are seen with ANTIthyroid drugs

A nurse is providing discharge teaching to a client who experienced DKA. Which of the following should the nurse include in the teaching? Select all that apply. A. Drink 3 L of fluid daily B. Monitor blood glucose every 4 hrs when ill C. Administer insulin as prescribed when ill D. Notify provider when BG is 200 mg/dL E. Report ketones in the urine after 24 hours of illness

A, B, C, E

A nurse is presenting information to a group of clients about nutrition habits that prevent type 2 diabetes mellitus. Which of the following should the nurse include in the information? Select all that apply. A. Eat less meat and processed foods. B. Decreased intake of saturated fat. C. Increase daily fiber intake. D. Limit saturated fat intake to 15% of daily caloric intake. E. Include omega-3 fatty acid in the diet

A, B, C, E Healthy nutrition should include decreasing consumption of meats and processed foods, which can prevent diabetes and hyperlipidemia. Healthy nutrition should include lowering LDL by decreasing intake of saturated fats. Healthy nutrition should include increasing dietary fiber to control weight gain and decrease the risk of diabetes and hyperlipidemia. Healthy nutrition should include omega-3 fatty acid for secondary prevention of diabetes and heart disease.

A nurse in a providers office is planning care for a client who has a new diagnosis of Graves' disease and a new prescription for methimazole (Tapazole). Which of the following should the nurse include in the plan of care? Select all that apply. A. Monitor CBC B. Monitor T3 C. Inform the client that the medication should not be taken for more than three months D. Advise the client to take the medication at the same time every day E. Inform the client that an adverse effects of this medication is iodine toxicity

A, B, D Methimazole can cause a number of hematologic effects, including leukopenia and thrombocytopenia. Therefore, the nurse should monitor the clients CBC. Methimazole reduce his thyroid hormone production. And it should be taken the same time every day to maintain blood levels.

A nurse is caring for a client who has primary diabetes insipidus (DI). Which of the following manifestations should the nurse expect to find? Select all that apply. A. Serum sodium of 155 mEq/L B. Fatigue C. Serum osmolality of 250 mOsm/L D. Polyuria E. Nocturia

A, B, D, E Primary DI is caused by a reduction in the secretion of antidiuretic hormone (ADH), which can result in an increased serum sodium. Can also result in fatigue due to electrolyte imbalance. Serum osmolality will be greater than 300 mOsm/L. Can also result in polyuria. Can also result in nocturia.

The nurse is monitoring a client receiving levothyroxine sodium, thyroid drug, for hypothyroidism. Which findings indicate the presence of a side effect associated with this medication? SELECT ALL THAT APPLY a. insomnia b. weight loss c. bradycardia d. constipation e. mild heat intolerance

A, B, E

The health care provider orders oral glucose tolerance testing for a patient seen in the clinic. Which information from the patient's health history is most important for the nurse to communicate to the health care provider? a. The patient had a viral illness 2 months ago. b. The patient uses oral contraceptives. c. The patient runs several days a week. d. The patient has a family history of diabetes.

B Rationale: Oral contraceptive use may falsely elevate oral glucose tolerance test (OGTT) values. A viral 2 months previously illness may be associated with the onset of type 1 diabetes but will not falsely impact the OGTT. Exercise and a family history of diabetes both can affect blood glucose but will not lead to misleading information from the OGTT.

Analyze the following diagnostic findings for your patient with type 2 diabetes. Which result will need further assessment? A) BP 126/80 B) A1C 9% C)FBG 130mg/dL D) LDL cholesterol 100mg/dL

B) A1C 9% Rationale: Lowering hemoglobin A1C (to average of 7%) reduces microvascular and neuropathic complications. Tighter glycemic control(normal A1C < 6%) may further reduce complications but increases hypoglycemia risk.

A home health nurse is at the home of a client with diabetes and arthritis. The client has difficulty drawing up insulin. It would be most appropriate for the nurse to refer the client to: A) A social worker from the local hospital B) An occupational therapist from the community center C) A physical therapist from the rehabilitation agency D) Another client with diabetes mellitus and takes insulin

B) An occupational therapist can assist a client to improve the fine motor skills needed to prepare an insulin injection.

A nurse is reviewing the clinical manifestations of hyperthyroidism with the client. Which of the following findings should the nurse include? Select all that apply. A. Dry skin B. Heat intolerance C. Constipation D. Palpitations E. Weight loss F. Bradycardia

B, D, E Hyperthyroidism increases the client's metabolism. Therefore, heat intolerance, palpitations, and weight loss are expected findings.

A client asks the nurse why the provider bases his medication regimen on his HbA1C instead of his log of morning fasting blood glucose results. Which of the following is an appropriate response by the nurse? A. HB A1C measures how well insulin is regulating your blood glucose between meals. B. HB A1 C indicates how well your blood glucose has been regulated over the past three months. C. A test of HB A1C is the first test to determine if an individual has diabetes. D. A test of HB A1C determines if the dosage of insulin needs to be adjusted.

B. HB A1C measures the client's BC control over the past 2 to 4 months.

A type 1 diabetic patient who was admitted with severe hypoglycemia and treated tells the nurse, "I did not have any of the usual symptoms of hypoglycemia." Which question by the nurse will help identify a possible reason for the patient's hypoglycemic unawareness? a. "Do you use any calcium-channel blocking drugs for blood pressure?" b. "Have you observed any recent skin changes?" c. "Do you notice any bloating feeling after eating?" d. "Have you noticed any painful new ulcerations or sores on your feet?"

C Rationale: Hypoglycemic unawareness is caused by autonomic neuropathy, which would also cause delayed gastric emptying. Calcium-channel blockers are not associated with hypoglycemic unawareness, although -adrenergic blockers can prevent patients from having symptoms of hypoglycemia. Skin changes can occur with diabetes, but these are not associated with autonomic neuropathy. If the patient can feel painful areas on the feet, neuropathy has not occurred.

The parathyroid glands produce PTH, which is important in the body as a. a modulator of the thyroid hormone b. a regulator of potassium c. a regulator of calcium d. an activator of vitamin D

C regulates calcium

The nurse is assessing a client who has SIADH. Which of the following findings indicate the client is experiencing a complication? A. Decreased central venous pressure (CVP) B. Increased urine output C. Distended neck veins D. Extreme thirst

C. Distended neck veins are a manifestation of fluid overload, which can lead to pulmonary edema and heart failure. Decreased CVP is indicative of shock. Increased urine output is indicative of DI. Extreme thirst is indicative of DI.

A nurse in a providers office is reviewing the laboratory findings of the client who's being evaluated for primary hypothyroidism. Which of the following laboratory findings is expected for a client who has hypothyroidism? A. Serum T4 10 mcg/dL B. Serum T3 200 ng/dL C. Hematocrit 34% D. Serum cholesterol 100 mg/dL

C. Hematocrit of 34% indicates anemia, which is an expected result for a client who has hypothyroidism.

A nurse is caring for a client admitted to the emergency department with diabetic ketoacidosis (DKA). In the acute phase, the priority nursing action is to prepare to: A. Correct the acidosis B. Administer 5% dextrose intravenously C. Administer regular insulin inraVenously D. Apply a monitor for an electrocardiogram.

C. Administer regular insulin inraVenously Lack (absolute or relative) of insulin is the primary cause of DKA. Treatment consists of insulin administration (regular insulin), intravenous fluid administration (normal saline initially), and potassium replacement, followed by correcting acidosis. Applying an electrocardiogram monitor is not a priority action.

An 18-year-old female client, 5'4'' tall, weighing 113 kg, comes to the clinic for a non-healing wound on her lower leg, which she has had for two weeks. Which disease process should the nurse suspect the client is developing? A. Type 1 diabetes B. Type 2 diabetes C. Gestational diabetes D. Acanthosis nigricans

CORRECT -->B. Type 2 diabetes is a disorder usually occurring around the age of 40, but it is now being detected in children and young adults as a result of obesity and sedentary lifestyles. Non-healing wounds are a hallmark sign of type 2 diabetes. This client weights 248.6 lbs and is short. A: Type 1 diabetes usually occurs in young clients who are underweight. In this disease, there is no production of insulin from the beta cells in the pancreas. People with type 1 diabetes are insulin dependent with a rapid onset of symptoms, including polyuria, polydipsia, and polyphagia. C. Gestational diabetes occurs during pregnancy. There is no mention of this. D. Acanthosis nigricans (AN), dark pigmentation and skin creases in the neck, is a sign of hyperinsulinemia. The pancreas is secreting excess amounts of insulin as a result of excessive caloric intake. It is identified in young children and is a precursor to the development of type 2 diabetes."

A hospitalized child has been diagnosed with SIADH (syndrome of inappropriate antidiuretic hormone), a complication of his meningitis. What would the nurse expect to see on this child's lab results? 1. Hyponatremia 2. Hypocalcemia 3. Hyperglycemia 4. Hypernatremia

Correct Answer: 1 Rationale 1: SIADH is associated with increased permeability in distal renal tubes, leading to water intoxication and low sodium. Rationale 2: Hypocalcemia is not seen with SIADH. Rationale 3: Hyperglycemia is not related to SIADH. Rationale 4: Hypernatremia is seen with DI, not SIADH.

A 12-year-old has been selected to be a cheerleader for her middle school. This child has been recently diagnosed with type 1 diabetes. In teaching this child's mother about care for her child, the nurse wants the mother to understand that with increased physical activity, the child will need: 1. Decreased food intake. 2. Increased doses of insulin. 3. Increased food intake. 4. Decreased doses of insulin.

Correct Answer: 3 Rationale 1: Decreased food intake would increase the chance of hypoglycemia. Rationale 2: Increased dose of insulin would cause hypoglycemia. Exercise causes insulin to be used more efficiently by the body, so an increase in insulin would not be needed. Rationale 3: An increase in physical activity requires an increase in caloric intake to prevent hypoglycemia. Rationale 4: A decreased dose of insulin would not allow the sugar to enter the cells where it is needed during exercise.

An adolescent female with untreated Graves' disease is admitted to the hospital. The nurse expects to find which signs and symptoms in this client? 1. Hyperglycemia, ketonuria, and glucosuria 2. Weight gain, hirsutism, and muscle weakness 3. Tachycardia, fatigue, and heat intolerance 4. Dehydration, metabolic acidosis, and hypertension

Correct Answer: 3 Rationale 1: Hyperglycemia, ketonuria, and glucosuria are signs of diabetes. Rationale 2: Weight gain, hirsutism, and muscle weakness are seen in clients with Cushing's disease. Rationale 3: Clinical manifestations of Graves' disease are tachycardia, fatigue, and heat intolerance, seen with hyperthyroidism. Rationale 4: Dehydration, metabolic acidosis, and hypertension are signs of congenital adrenal hyperplasia.

A nurse shoud recognize which symptom as a cardinal sign of diabetes mellitus? a. Nausea b. Seizure c. Hyperactivity d. Frequent urination

D. Frequent Urination Polyphagia, polyuria, polydipsia, and weight loss are cardinal signs of DM. Other signs include irritability, shortened attention span, lowered frustration tolerance, fatigue, dry skin, blurred vision, sores that are slow to heal, and flushed skin.

The benefits of using an insulin pump include all of the following except: "a. By continuously providing insulin they eliminate the need for injections of insulin b. They simplify management of blood sugar and often improve A1C c. They enable exercise without compensatory carbohydrate consumption d. They help with weight loss

D: Using an insulin pump has many advantages, including fewer dramatic swings in blood glucose levels, increased flexibility about diet, and improved accuracy of insulin doses and delivery; however, the use of an insulin pump has been associated with weight gain.

A client is evaluated for type 1 diabetes. Which client comment correlates best with this disorder? a) "I have a cough and cold that just won't go away." b) "I'm thirsty all the time. I just can't get enough to drink." c) "I notice pain when I urinate." d) "It seems like I have no appetite. I have to make myself eat."

I'm thirsty all the time. I just can't get enough to drink." Explanation: Classic signs and symptoms of diabetes mellitus are polydipsia (excessive thirst), polyuria (excessive urination), and polyphagia (excessive appetite). Decreased appetite, lingering cough and cold, and pain on urination aren't related to diabetes. Decreased appetite reflects a GI disorder; cough and cold indicate an upper respiratory problem; and pain on urination suggests a urinary tract infection.

The nurse is educating a group of older clients at a senior center about manifestations of diabetes mellitus. Which manifestations should the nurse include about diabetes mellitus that may be altered because of the aging​ process? Select all that apply. a. Decreased thirst b. Increased hunger c. Hypotension d. Peripheral neuropathy e. Urinary incontinence

a. Decreased thirst d. Peripheral neuropathy e. Urinary incontinence

A patient recently diagnosed with type 1 diabetes mellitus does not understand why the disease developed because the patient is thin and eats all of the time. What is the most appropriate response by the​ nurse? a. ​"Diabetes makes it difficult for your body to obtain energy from the foods you​ eat." b. ​"Your condition makes it impossible for you to gain​ weight." c. ​"Thin people can be​ diabetic, too." d. ​"Your lab tests indicate the presence of​ diabetes."

a. ​"Diabetes makes it difficult for your body to obtain energy from the foods you​ eat."

The nurse is identifying patients at risk for needing insulin. Which patients should the nurse identify as potentially needing insulin to maintain a normal blood glucose​ level? Select all that apply. a. patients receiving total parenteral nutrition b. patients with type 2 diabetes who are undergoing surgical procedures c. patients who are fasting or malnourished d. patients with type 2 diabetes who are diagnosed with an infection e. patients with gestational diabetes

b. patients with type 2 diabetes who are undergoing surgical procedures d. patients with type 2 diabetes who are diagnosed with an infection e. patients with gestational diabetes

A patient with hypothyroidism is treated with Synthroid. When teaching the patient about the therapy, the nurse a. explains that caloric intake must be reduced when drug therapy is started b. provides written instruction for all information related to the medication therapy c. assures the patient that a return to normal function will occur with replacement therapy d. informs the patient that medications must be taken until hormone balance is reestablished

b. provides written instruction for all information related to the medication therapy (rationale- because of the mental sluggishness, inattentiveness, and memory loss that occur with hypothyroidism, it is important to provide written instructions and repeat information when teaching the patient. Caloric intake can be increased when drug therapy is started, because of an increased metabolic rate, and replacement therapy must be taken for life. Although most patients return to a normal state with treatment, cardiovascular conditions and psychoses may persist.)

A patient with type 1 diabetes mellitus has difficulty swallowing and takes milk of magnesium every day for nausea and constipation. What should the nurse suspect is occurring with this​ patient? a. reaction to insulin injections b. visceral neuropathy c. peripheral neuropathy ​d. age-related changes

b. visceral neuropathy

The nurse is reviewing instruction provided to a graduate nurse regarding insulin therapy. Which statement made by the graduate indicates that further instruction is​ needed? Select all that apply. a. Regular insulin can be administered intravenously. b. Lispro is a​ rapid-acting insulin. c. Insulin detemir is administered prior to each meal. d. NPH insulin may be mixed with lispro insulin. e. Insulin glargine may be used to treat gestational diabetes.

c. Insulin detemir is administered prior to each meal. e. Insulin glargine may be used to treat gestational diabetes.

The nurse is working with an overweight client who has a high-stress job and smokes. This client has just received a diagnosis of Type II Diabetes and has just been started on an oral hypoglycemic agent. Which of the following goals for the client which if met, would be most likely to lead to an improvement in insulin efficiency to the point the client would no longer require oral hypoglycemic agents? a. Comply with medication regimen 100% for 6 months b. Quit the use of any tobacco products by the end of three months c. Lose a pound a week until weight is in normal range for height and exercise 30 minutes daily d. Practice relaxation techniques for at least five minutes five times a day for at least five months

c. Lose a pound a week until weight is in normal range for height and exercise 30 minutes daily When type II diabetics lose weight through diet and exercise they sometimes have an improvement in insulin efficiency sufficient to the degree they no longer require oral hypoglycemic agents.

A patient is admitted to the hospital in thyrotoxic crisis. On physical assessment of the patient, the nurse would expect to find a. hoarseness and laryngeal stridor b. bulging eyeballs and arrhythmias c. elevated temperature and signs of heart failure d. lethargy progressing suddenly to impairment of consciousness

c. elevated temperature and signs of heart failure (rationale- a hyperthyroid crisis results in marked manifestations of hyperthyroidism, with fever tachycardia, heart failure, shock, hyperthermia, agitation, N/V/D, delirium, and coma. Although exophthalmos may be present in the patient with Gravs' dz, it is not a significant factor in hyperthyroid crisis. Hoarsness and laryngeal stridor are characteristic of the tetany of hypoparathyroidism, and lethargy progressing to coma is characteristic of myxedema coma, a complication of hypothyroidism.

The nurse has​ completed educating a client diagnosed with type 1 diabetes mellitus about​ medication, nutrition, and exercise. What statement by the client indicates the client has a good understanding of how to properly treat diabetes​ mellitus? a. ​"I will take oral hypoglycemic agents to control my blood​ glucose." ​b. "I need to exercise at least 120 minutes each​ week." c. ​"I need to learn sick day management​ rules." d. ​"I will prevent prolonged sessions of exercise because they cause​ hyperglycemia."

c. ​"I need to learn sick day management​ rules."

Which of the following best describes the pathophysiology of Type 1 diabetes​ mellitus? a. Production of pancreatic beta cells b. Production of pancreatic alpha cells c. Destruction of pancreatic alpha cells d. Destruction of pancreatic beta cells

d. Destruction of pancreatic beta cells

A patient with type 2 diabetes mellitus is scheduled for laparoscopic adjustable gastric banding​ (LAGB) surgery. What should the nurse explain to the patient about this procedure and​ diabetes? a. ​"Surgical procedures can be dangerous for patients with​ diabetes." b. ​"This procedure is more appropriate for a patient who has a diagnosis of type 1 diabetes​ mellitus." c. ​"Do you feel that a surgical weight loss procedure will cure your​ obesity?" d. ​"Evidence indicates positive outcomes for many patients with diabetes who undergo surgical weight loss​ procedures."

d. ​"Evidence indicates positive outcomes for many patients with diabetes who undergo surgical weight loss​ procedures."

A patient at risk for the development of type 2 diabetes mellitus asks why weight loss will reduce risk of the condition. Which response by the nurse is most​ accurate? a. ​"The physical inactivity associated with obesity causes a reduced ability by the body to produce​ insulin." b. ​"The amount of foods taken in require more insulin to adequately metabolize​ them, resulting in​ diabetes." c. ​"Thin people are less likely to become​ diabetic." d. ​"Excess body weight impairs the​ body's release of​ insulin."

d. ​"Excess body weight impairs the​ body's release of​ insulin."

The nurse is planning care for a patient with type 1 diabetes mellitus. Which action should the nurse identify as being the most effective to reduce the development of​ complications? a. the necessity of a yearly eye exam b. knowing symptoms of urinary tract infections c. performance of effective foot care d. ​self-monitoring of blood glucose levels

d. ​self-monitoring of blood glucose levels


Conjuntos de estudio relacionados

Exam 1 - 5020; Foundations & Theory

View Set

DECA Marketing Cluster Sample Exam #1 of 5

View Set

CHS Sociology Ch. 10 (Social Institutions) Review

View Set

EPS 601 Chapter 9 (Behavior therapy, study guide)

View Set

PrepU | Assignment 10 | Chapter 40: Fluid, Electrolyte, and Acid-Base Balance

View Set

ch. 13 Strict Liability and Product Liability

View Set